Série de Fourier em cossenos.  matemática superior

Série de Fourier em cossenos. matemática superior

transcrição

1 MINISTÉRIO DA EDUCAÇÃO E CIÊNCIA DA FEDERAÇÃO RUSSA FACULDADE DE FÍSICA DA UNIVERSIDADE ESTADUAL DE NOVOSIBIRSK R. K. Belkheeva SÉRIE FOURIER EM EXEMPLOS E TAREFAS Tutorial Novosibirsk 211

2 UDC BBK V161 B44 B44 Belkheeva R. K. Série de Fourier em exemplos e problemas: Textbook / Novosib. Estado un-t. Novosibirsk, s. ISBN B guia de estudo as informações básicas sobre a série de Fourier são apresentadas, exemplos são dados para cada tópico estudado. Um exemplo de aplicação do método de Fourier para resolver o problema das vibrações transversais de uma corda é analisado em detalhes. Material ilustrativo é fornecido. Existem tarefas para solução independente. Destina-se a estudantes e professores da Faculdade de Física da Universidade Estadual de Novosibirsk. Publicado de acordo com a decisão da Comissão Metodológica da Faculdade de Física da NSU. Revisor Dr. phys.-matemática. Ciências. V. A. Aleksandrov ISBN c Novosibirsk State University, 211 c Belkheeva R. K., 211

3 1. Expansão em série de Fourier de uma função 2π-periódica Definição. A série de Fourier da função f(x) é a série funcional a 2 + (a n cosnx + b n sin nx), (1) onde os coeficientes a n, b n são calculados pelas fórmulas: a n = 1 π b n = 1 π f (x) cosnxdx, n = , 1,..., (2) f(x) sin nxdx, n = 1, 2,.... (3) Fórmulas (2) (3) são chamadas de fórmulas de Euler Fourier . O fato de que a função f(x) corresponde à série de Fourier (1) é escrita como uma fórmula f(x) a 2 + (a n cosnx + b n sin nx) (4) e eles dizem que o lado direito da fórmula ( 4) é uma série formal de funções de Fourier f(x). Em outras palavras, a fórmula (4) significa apenas que os coeficientes a n, b n são encontrados pelas fórmulas (2), (3). 3

4 Definição. Uma função 2π-periódica f(x) é chamada de suave por partes se o intervalo [, π] contiver um número finito de pontos = x< x 1 <... < x n = π таких, что в каждом открытом промежутке (x j, x j+1) функция f(x) непрерывно дифференцируема, а в каждой точке x j существуют конечные пределы слева и справа: f(x j) = lim h + f(x j h), f(x j +) = lim h + f(x j + h), (5) f(x j h) f(x j) f(x j + h) f(x j +) lim, lim. h + h h + h (6) Отметим, что последние два предела превратятся в односторонние производные после замены предельных значений f(x j) и f(x j +) значениями f(x j). Теорема о представимости кусочно-гладкой функции в точке своим рядом Фурье (теорема о поточечной сходимости). Ряд Фурье кусочно-гладкой 2π-периодической функции f(x) сходится в каждой точке x R, а его сумма равна числу f(x), если x точка непрерывности функции f(x), f(x +) + f(x) и равна числу, если x точка разрыва 2 функции f(x). ПРИМЕР 1. Нарисуем график, найдем ряд Фурье функции, заданной на промежутке [, π] формулой, f(x) = x, предполагая, что она имеет период 2π, и вычислим суммы 1 1 числовых рядов (2n + 1) 2, n 2. n= Решение. Построим график функции f(x). Получим кусочно-линейную непрерывную кривую с изломами в точках x = πk, k целое число (рис. 1). 4

5 Fig. 1. Gráfico da função f(x) nx + π n n 2 = 2 π (1) n 1 n 2 = b n = 1 π π = 2 π f(x) cosnxdx = cos nx cos n 2 = 4 πn2, para n ímpar, para n par, f(x) sen nxdx = porque a função f(x) é par. Escrevemos a série formal de Fourier para a função f(x): f(x) π 2 4 π k= 5 cos (2k + 1)x (2k + 1) 2.

6 Descubra se a função f(x) é suave por partes. Como é contínua, calculamos apenas os limites (6) nos pontos finais do intervalo x = ±π e no ponto de quebra x = : e f(π h) f(π) π h π lim = lim h + h h + h = 1, f(+ h) f(+) + h () lim = lim h + h h + h f(+ h) f(+) + h lim = lim = 1, h + h h + h = 1 , f(h) f () h () lim = lim = 1. h + h h + h Os limites existem e são finitos, portanto a função é suave por partes. Pelo teorema da convergência pontual, sua série de Fourier converge para o número f(x) em cada ponto, ou seja, f(x) = π 2 4 π k= cos (2k + 1) + x (2k + 1) 2 = = π 2 4 (cosx + 19 π cos 3x) cos 5x (7) As Figuras 2 e 3 mostram o caráter da aproximação das somas parciais da série de Fourier S n (x), onde S n (x) = a n 2 + (a k coskx + b k sin kx), k=1, para a função f(x) no intervalo [, π] . 6

7 Fig. Fig. 2. Gráfico da função f(x) com gráficos sobrepostos de somas parciais S (x) = a 2 e S 1(x) = a 2 + a 1 cos x 3. Gráfico da função f (x) com um gráfico de soma parcial sobreposto a ele S 99 (x) \u003d a 2 + a 1 cos x + + a 99 cos 99x 7

8 Substituindo em (7) x = obtemos: = π 2 4 π k= 1 (2k + 1) 2, de onde encontramos a soma da série numérica: = π2 8. Conhecendo a soma desta série, é fácil encontrar a seguinte soma Temos: S = ( ) S = ()= π S, portanto S = π2 6, ou seja, 1 n = π A soma desta famosa série foi encontrada pela primeira vez por Leonhard Euler. É frequentemente encontrado em análise matemática e suas aplicações. EXEMPLO 2. Desenhe um gráfico, encontre a série de Fourier da função dada pela fórmula f(x) = x para x< π, предполагая, что она имеет период 2π, и вычислим суммы числовых (1) n) рядов + n= ((2n + 1,) (k k + 1) Решение. График функции f(x) приведен на рис. 4. 8

9 Fig. 4. Gráfico da função f(x) A função f(x) é continuamente diferenciável no intervalo (, π). Nos pontos x = ±π, tem limites finitos (5): f() =, f(π) = π. Além disso, existem limites finitos (6): f(+ h) f(+) lim = 1 e h + h f(π h) f(π +) lim = 1. h + h Portanto, f(x) é função suave por partes. Como a função f(x) é ímpar, então a n =. Os coeficientes b n são encontrados por integração por partes: b n = 1 π f(x) sen πnxdx= 1 [ x cosnx π πn + 1 n = 1 πn [(1)n π + (1) n π] = 2(1 )n+ um. n Vamos compor a série formal de Fourier da função 2(1) n+1 f(x) sen nx. n 9 cosnxdx ] =

10 De acordo com o teorema da convergência pontual para uma função 2π-periódica suave por partes, a série de Fourier da função f(x) converge para a soma: 2(1) n+1 sen nx = n f(x) = x se π< x < π, = f(π) + f(π +) 2 =, если x = π, (8) f() + f(+) =, если x =. 2 На рис. 5 8 показан характер приближения частичных сумм S n (x) ряда Фурье к функции f(x). Рис. 5. График функции f(x) с наложенным на него графиком частичной суммы S 1 (x) = a 2 + a 1 cos x 1

11 Fig. Fig. 6. Gráfico da função f(x) com o gráfico da soma parcial S 2 (x) sobreposta a ela. 7. Gráfico da função f(x) com o gráfico da soma parcial S 3 (x) 11 sobreposta a ela

12 Fig. 8. Gráfico da função f(x) com o gráfico da soma parcial S 99 (x) sobreposto a ela Usamos a série de Fourier obtida para encontrar as somas de duas séries numéricas. Colocamos em (8) x = π/2. Então 2 () +... = π 2, ou = n= (1) n 2n + 1 = π 4. Encontramos facilmente a soma da conhecida série de Leibniz. Colocando x = π/3 em (8), encontramos () +... = π 2 3, ou (1+ 1) () (k) 3π +...= 3k

13 EXEMPLO 3. Desenhe um gráfico, encontre a série de Fourier da função f(x) = sen x, supondo que ela tenha um período de 2π, e 1 calcule a soma da série numérica 4n 2 1. Solução. O gráfico da função f(x) é mostrado na fig. 9. Obviamente, f(x) = sen x é uma função contínua par com período π. Mas 2π é também o período da função f(x). Arroz. 9. Gráfico da função f(x) Vamos calcular os coeficientes de Fourier. Todos b n = porque a função é par. Usando fórmulas trigonométricas, calculamos a n para n 1: a n = 1 π = 1 π sen x cosnxdx = 2 π sen x cosnxdx = (sin(1 + n)x sen(1 n)x) dx = = 1 () π cos( 1 + n)x cos(1 n)x + = 2 () 1 + (1) n = π 1 + n 1 n π 1 n 2 ( 4 1 se n = 2k, = π n 2 1 se n = 2k

14 Este cálculo não permite encontrar o coeficiente a 1 porque em n = 1 o denominador vai a zero. Portanto, calculamos o coeficiente a 1 diretamente: a 1 = 1 π sen x cosxdx =. Como f(x) é continuamente diferenciável em (,) e (, π) e nos pontos kπ, (k é um inteiro), existem limites finitos (5) e (6), a série de Fourier da função converge para em todos os pontos: = 2 π 4 π sinx = 2 π 4 π cos 2nx 4n 2 1 = (1 1 cos 2x cos 4x + 1) cos 6x 1. Gráfico da função f(x) com o gráfico da soma parcial S(x) sobreposta a ela 14

15 Fig. Fig. 11. Gráfico da função f(x) com o gráfico da soma parcial S 1 (x) sobreposta a ela. Fig. 12. Gráfico da função f(x) com o gráfico da soma parcial S 2 (x) sobreposta a ela. 13. Gráfico da função f(x) com o gráfico da soma parcial S 99 (x) 15 sobreposta a ela

16 1 Calcule a soma da série numérica. Para fazer isso, colocamos 4n 2 1 em (9) x =. Então cosnx = 1 para todo n = 1, 2,... e Portanto, 2 π 4 π 1 4n 2 1 =. 1 4n 2 1 = = 1 2. EXEMPLO 4. Vamos provar que se uma função contínua suave por partes f(x) satisfaz a condição f(x π) = f(x) para todo x (ou seja, é π-periódica) , então a 2n 1 = b 2n 1 = para todo n 1, e vice-versa, se a 2n 1 = b 2n 1 = para todo n 1, então f(x) é π-periódica. Solução. Seja a função f(x) π-periódica. Vamos calcular seus coeficientes de Fourier a 2n 1 e b 2n 1: = 1 π (a 2n 1 = 1 π f(x) cos(2n 1)xdx + f(x) cos(2n 1)xdx =) f(x) ) cos (2n 1)xdx. Na primeira integral fazemos a mudança de variável x = t π : f(x) cos(2n 1)xdx = f(t π) cos(2n 1)(t + π) dt. 16

17 Usando o fato de que cos(2n 1)(t + π) = cos(2n 1)t ef(t π) = f(t), obtemos: a 2n 1 = 1 π (f(x) cos( 2n 1)x dx+) f(x) cos(2n 1)x dx =. Prova-se igualmente que b 2n 1 =. Inversamente, seja a 2n 1 = b 2n 1 =. Como a função f(x) é contínua, então, pelo teorema da representabilidade de uma função em um ponto por sua série de Fourier, temos Então f(x π) = f(x) = (a 2n cos 2nx + b 2n sen 2nx). (a2n cos 2n(x π) + b 2n sen 2n(x π)) = (a2n cos 2nx + b 2n sen 2nx) = f(x), o que significa que f(x) é uma função π-periódica. EXEMPLO 5. Vamos provar que se uma função suave por partes f(x) satisfaz a condição f(x) = f(x) para todo x, então a = e a 2n = b 2n = para todo n 1, e vice-versa , se a = a 2n = b 2n =, então f(x π) = f(x) para todo x. Solução. Deixe que a função f(x) satisfaça a condição f(x π) = f(x). Vamos calcular seus coeficientes de Fourier: 17

18 = 1 π (an = 1 π f(x) cos nxdx + f(x) cosnxdx =) f(x) cosnxdx. Na primeira integral fazemos a mudança de variável x = t π. Então f(x) cosnxdx = f(t π) cosn(t π) dt. Usando o fato de que cos n(t π) = (1) n cosnt ef(t π) = f(t), obtemos: a n = 1 π ((1) n) f(t) cosnt dt = if n par, = 2 π f(t) cos nt dt, se n for ímpar. π Prova-se igualmente que b 2n =. Inversamente, seja a = a 2n = b 2n =, para todo n 1. Como a função f(x) é contínua, então, pelo teorema da representabilidade de uma função em um ponto, sua série de Fourier satisfaz a igualdade f( x) = (a 2n 1 cos ( 2n 1)x + b 2n 1 sen (2n 1)x). dezoito

19 Então = f(x π) = = = f(x). EXEMPLO 6. Vamos estudar como estender a função f(x) integrável no intervalo [, π/2] ao intervalo [, π], de modo que sua série de Fourier tenha a forma: a 2n 1 cos(2n 1) x. (1) Solução. Deixe o gráfico da função ter a forma mostrada na Fig. 14. Como na série (1) a = a 2n = b 2n = para todo n, segue do Exemplo 5 que a função f(x) deve satisfazer a igualdade f(x π) = f(x) para todo x. Esta observação permite estender a função f(x) ao intervalo [, /2] : f(x) = f(x+π), fig. 15. Do fato de que a série (1) contém apenas cossenos, concluímos que a função contínua f (x) deve ser par (ou seja, seu gráfico deve ser simétrico em relação ao eixo Oy), Fig.

20 Fig. 14. Gráfico da função f(x) 15. Gráfico da continuação da função f(x) no intervalo [, /2] 2

21 Assim, a função desejada tem a forma mostrada na fig. 16. Fig. 16. Gráfico da continuação da função f(x) no intervalo [, π] Resumindo, concluímos que a função deve ser continuada da seguinte forma: f(x) = f(x), f(π x) = f(x), ou seja, intervalo [π/2, π], o gráfico da função f(x) é centralmente simétrico em relação ao ponto (π/2,), e no intervalo [, π], seu gráfico é simétrica em relação ao eixo Oy. 21

22 GENERALIZAÇÃO DOS EXEMPLOS 3 6 Seja l >. Considere duas condições: a) f(l x) = f(x); b) f(l + x) = f(x), x [, l/2]. Do ponto de vista geométrico, a condição (a) significa que o gráfico da função f(x) é simétrico em relação à linha vertical x = l/2, e a condição (b) que o gráfico f(x) é centralmente simétrica em relação ao ponto (l/2;) no eixo das abcissas. Então as seguintes afirmações são verdadeiras: 1) se a função f(x) for par e a condição (a) for satisfeita, então b 1 = b 2 = b 3 =... =, a 1 = a 3 = a 5 = ... = ; 2) se a função f(x) for par e a condição (b) for satisfeita, então b 1 = b 2 = b 3 =... =, a = a 2 = a 4 =... = ; 3) se a função f(x) for ímpar e a condição (a) for satisfeita, então a = a 1 = a 2 =... =, b 2 = b 4 = b 6 =... = ; 4) se a função f(x) for ímpar e a condição (b) for satisfeita, então a = a 1 = a 2 =... =, b 1 = b 3 = b 5 =... =. PROBLEMAS Nos problemas 1 7 desenhe gráficos e encontre a série de Fourier para as funções, (assumindo que elas tenham um período de 2π: se< x <, 1. f(x) = 1, если < x < π. 1, если < x < /2, 2. f(x) =, если /2 < x < π/2, 1, если π/2 < x < π. 3. f(x) = x 2 (< x < π). 4. f(x) = x 3 (< x < π). { π/2 + x, если < x <, 5. f(x) = π/2 x, если < x < π. 22

23 (1 se /2< x < π/2, 6. f(x) = 1, если π/2 < x < 3π/2. {, если < x <, 7. f(x) = sin x, если < x < π. 8. Как следует продолжить интегрируемую на промежутке [, π/2] функцию f(x) на промежуток [, π], чтобы ее ряд Фурье имел вид: b 2n 1 sin (2n 1)x? Ответы sin(2n 1)x sin(2n + 1)x. π 2n 1 π 2n + 1 n= 3. 1 (1) n () 12 3 π2 + 4 cosnx. 4. (1) n n 2 n 2π2 sin nx. 3 n 5. 4 cos(2n + 1)x π (2n + 1) (1) n cos(2n + 1)x. π 2n + 1 n= n= 7. 1 π sin x 2 cos 2nx. 8. Функцию следует продолжить следующим образом: f(x) = f(x), f(π x) = f(x), π 4n 2 1 то есть на промежутке [, π], график функции f(x) будет симметричен относительно вертикальной прямой x = π/2, на промежутке [, π] ее график центрально симметричен относительно точки (,). 23

24 2. Expansão de uma função dada no intervalo [, π] apenas em termos de senos ou apenas em termos de cossenos Seja uma função f dada no intervalo [, π]. Para expandi-lo neste intervalo em uma série de Fourier, primeiro estendemos f no intervalo [, π] de forma arbitrária, e então usamos as fórmulas de Euler Fourier. A arbitrariedade na continuação de uma função leva ao fato de que para a mesma função f: [, π] R podemos obter diferentes séries de Fourier. Mas é possível usar essa arbitrariedade de forma a obter uma expansão apenas em senos ou apenas em cossenos: no primeiro caso, basta continuar f de maneira ímpar, e no segundo, de maneira par. Algoritmo de solução 1. Continue a função de maneira ímpar (par) em (,), e então periodicamente com um período de 2π continue a função em todo o eixo. 2. Calcule os coeficientes de Fourier. 3. Componha a série de Fourier da função f(x). 4. Verifique as condições de convergência das séries. 5. Especifique a função para a qual essa série convergirá. EXEMPLO 7. Expanda a função f(x) = cosx,< x < π, в ряд Фурье только по синусам. Решение. Продолжим функцию нечетным образом на (,) (т. е. так, чтобы равенство f(x) = f(x) выполнялось для всех x (, π)), а затем периодически с периодом 2π на всю ось. Получим функцию f (x), график которой приведен на рис

25 Fig. 17. Gráfico da função continuada Obviamente, a função f(x) é suave por partes. Vamos calcular os coeficientes de Fourier: a n = para todo n porque a função f (x) é ímpar. Se n 1, então b n = 2 π f(x) sen πnxdx = 2 π cosx sen nxdx = = 2 π dx = = 2 π cos (n + 1) x cos (n 1) x + = π n + 1 n 1 = 1 (1) n (1)n 1 1 = π n + 1 n 1 = 1 se n = 2 k + 1, (1)n+1 (n 1) + (n + 1) = π ( n + 1)(n 1) 2 2n se n = 2k. π n 2 1 Para n = 1 nos cálculos anteriores, o denominador se anula, então o coeficiente b 1 pode ser calculado diretamente.

26 Essencialmente: b 1 = 2 π cosx sen xdx =. Componha a série de Fourier da função f (x): f (x) 8 π k=1 k 4k 2 1 sen 2kx. Como a função f (x) é suave por partes, então, pelo teorema da convergência pontual, a série de Fourier da função f (x) converge para a soma cosx se π< x <, S(x) =, если x =, x = ±π, cosx, если < x < π. В результате функция f(x) = cosx, заданная на промежутке (, π), выражена через синусы: cosx = 8 π k=1 k 4k 2 1 sin 2kx, x (, π). Рис демонстрируют постепенное приближение частичных сумм S 1 (x), S 2 (x), S 3 (x) к разрывной функции f (x). 26

27 Fig. Fig. 18. Gráfico da função f (x) com o gráfico da soma parcial S 1 (x) sobreposta a ela. 19. Gráfico da função f(x) com o gráfico da soma parcial S 2 (x) sobreposta a ela 27

28 Fig. Fig. 2. Gráfico da função f (x) com o gráfico da soma parcial S 3 (x) sobreposta a ela. 21 mostra gráficos da função f (x) e sua soma parcial S 99 (x). Arroz. 21. Gráfico da função f (x) com um gráfico da soma parcial S 99 (x) 28 sobreposta a ela

29 EXEMPLO 8. Vamos expandir a função f(x) = e ax, a >, x [, π], em uma série de Fourier apenas em cossenos. Solução. Continuamos a função de maneira uniforme para (,) (ou seja, de modo que a igualdade f(x) = f(x) vale para todo x (, π)), e então periodicamente com um período de 2π para todo o real eixo. Obtemos a função f (x), cujo gráfico é mostrado na Fig. 22. Função f (x) em pontos 22. O gráfico da função contínua f (x) x = kπ, k é um número inteiro, tem torções. Calculemos os coeficientes de Fourier: b n =, pois f (x) é par. Integrando por partes, obtemos 29

30 a n = 2 π a = 2 π = 2 cosnxd(e ax) = 2 πa e ax dx = 2 π a (eaπ 1), f(x) cos πnxdx = 2 π πa eax cosnx = 2 πa (eaπ cosnπ 1 ) + 2n πa 2 π e ax cos nxdx = + 2n e ax sen nxdx = πa sen nxde ax = = 2 π a (eaπ cos n π 1) + 2n π sen nx π a 2eax 2n2 e ax cos nxdx = 2 π a 2 π a (eaπ cos n π 1) n2 a a n. 2 Portanto, a n = 2a e aπ cos n π 1. π a 2 + n 2 Como f (x) é contínua, de acordo com o teorema da convergência pontual, sua série de Fourier converge para f (x). Assim, para todo x [, π] temos f(x) = 1 π a (eaπ 1)+ 2a π k=1 e aπ (1) k 1 a 2 + k 2 coskx (x π). As figuras demonstram a aproximação gradual das somas parciais da série de Fourier a uma dada função descontínua. 3

31 Fig. 23. Gráficos de funções f (x) e S (x) 24. Gráficos de funções f (x) e S 1 (x) 25. Gráficos de funções f (x) e S 2 (x) 26. Gráficos de funções f (x) e S 3 (x) 31

32 Fig. 27. Gráficos de funções f (x) e S 4 (x) 28. Gráficos das funções f (x) e S 99 (x) PROBLEMA 9. Expanda a função f (x) = cos x, x π, em uma série de Fourier apenas em cossenos. 1. Expanda a função f (x) \u003d e ax, a >, x π, em uma série de Fourier apenas em termos de senos. 11. Expanda a função f (x) \u003d x 2, x π, em uma série de Fourier apenas em senos. 12. Expanda a função f (x) \u003d sin ax, x π, em uma série de Fourier apenas em termos de cossenos. 13. Expanda a função f (x) \u003d x sin x, x π, em uma série de Fourier apenas em senos. Respostas 9. cosx = cosx. 1. e ax = 2 [ 1 (1) ke aπ] k sin kx. π a 2 + k2 k=1 11. x 2 2 [ π 2 (1) n 1 π n + 2 ] n 3 ((1)n 1) sen nx. 32

33 12. Se a não for um número inteiro, então sin ax = 1 cosaπ (1 + +2a cos 2nx ) + π a 2 (2n) 2 +2a 1 + cosaπ cos(2n 1)x π a 2 (2n 1) 2; se a = 2m é um número par, então sen 2mx = 8m cos(2n 1)x π (2m) 2 (2n 1) 2; se a = 2m 1 é um número ímpar positivo, então sin(2m 1)x = 2 (cos 2nx) 1 + 2(2m 1). π (2m 1) 2 (2n) π 16 n sen x sen 2nx. 2 π (4n 2 1) 2 3. Série de Fourier de uma função com período arbitrário Suponha que a função f(x) é definida no intervalo [ l, l], l >. Substituindo x = ly, y π, obtemos a função g(y) = f(ly/π) definida no intervalo π [, π]. Esta função g(y) corresponde à série (formal) de Fourier () ly f = g(y) a π 2 + (a n cosny + b n sin ny), cujos coeficientes são encontrados pelas fórmulas de Euler Fourier: a n = 1 π g(y) cosny dy = 1 π f (ly π) cos ny dy, n =, 1, 2,..., 33

34 b n = 1 π g(y) sinny dy = 1 π f () ly sin ny dy, n = 1, 2,.... π l, obtemos uma série trigonométrica ligeiramente modificada para a função f(x): onde f(x) a 2 + a n = 1 l b n = 1 l l l l l (a n cos πnx l f(x) cos πnx l f(x) sen πnx l + b n sen πnx), (11) l dx, n =, 1, 2 ,..., (12) dx, n = 1, 2,.... (13) As fórmulas (11) (13) definem a expansão em uma série de Fourier de uma função com um período arbitrário. EXEMPLO 9. Encontre a série de Fourier da função dada no intervalo (l, l) pela expressão (A se l< x, f(x) = B, если < x < l, считая, что она периодична с периодом 2l. Решение. Продолжим функцию периодически, с периодом 2l, на всю ось. Получим функцию f (x), кусочно-постоянную в промежутках (l + 2kl, l + 2kl), и претерпевающую разрывы первого рода в точках x = lk, k целое число. Ее коэффициенты Фурье вычисляются по формулам (12) и (13): 34

35 a = 1 l l f(x) dx = 1 l A dx + 1 l l B dx = A + B, l l a n = 1 l l l f(x) cos πnx l dx = = 1 l = 1 l l A cos πnx l = A + B π n l b n = 1 l dx + 1 l l B cos πnx l sen πn = se n, l l A sen πnx l f(x) sen πnx l dx + 1 l l dx = B sen πnx l = BA (1 cosπn). πn Componha a série de Fourier da função f (x) : f(x) A + B π (BA Desde cosπn = (1) n, então n dx = dx = (1 cosπn) sen πnx). l para n = 2k obtemos b n = b 2k =, para n = 2k 1 b n = b 2k 1 = 35 2(BA) π(2k 1).

36 Portanto f(x) A + B (BA) π (sen πx + 1 3πx sen + 1 5πx sen +... l 3 l 5 l De acordo com o teorema da convergência pontual, a série de Fourier da função f(x) converge para a soma A, se l< x, S(x) = A + B, если x =, x = ±l, 2 B, если < x < l. Придавая параметрам l, A, B конкретные значения получим разложения в ряд Фурье различных функций. Пусть l = π, A =, B = 3π. На рис. 29 приведены графики первых пяти членов ряда, функции f (x) и частичной суммы S 7 (x) = a 2 + b 1 sin x b 7 sin 7x. Величина a является средним значением функции на промежутке. Обратим внимание на то, что с возрастанием ча- 2 стоты гармоники ее амплитуда уменьшается. Для наглядности графики трех высших гармоник сдвинуты по вертикали. На рис. 3 приведен график функции f(x) и частичной суммы S 99 (x) = a 2 + b 1 sin x b 99 sin 99x. Для наглядности на рис. 31 приведен тот же график в другом масштабе. Последние два графика иллюстрируют явление Гиббса. 36).

37 Fig. 29. Gráfico da função f (x) com gráficos sobrepostos dos harmônicos S (x) = a 2 e S 1 (x) = b 1 sinx. Para maior clareza, os gráficos dos três harmônicos superiores S 3 (x) \u003d b 3 sin 3πx, S l 5 (x) \u003d b 5 sin 5πx l e S 7 (x) \u003d b 7 sin 7πx são deslocados verticalmente até 37

38 Fig. Fig. 3. Gráfico da função f(x) com o gráfico da soma parcial S 99 (x) sobreposta a ela. 31. Fragmento da fig. 3 em outra escala 38

39 PROBLEMAS Em problemas, expanda as funções especificadas na série de Fourier em intervalos dados. 14. f(x) = x 1, (1, 1). 15. f(x) = ch2x, (2, 2] f(x) = x (1 x), (1, 1]. 17. f(x) = cos π x, [ 1, 1] f(x) ) = sin π x, (1, 1).( 2 1 se 1< x < 1, 19. f(x) = 2l = 4., если 1 < x < 3; x, если x 1, 2. f(x) = 1, если 1 < x < 2, 2l = 3. { 3 x, если 2 x < 3;, если ωx, 21. f(x) = 2l = 2π/ω. sin ωx, если ωx π; Разложить в ряды Фурье: а) только по косинусам; б) только по синусам указанные функции в заданных промежутках (, l) { 22. f(x) = { 23. f(x) = ax, если < x < l/2, a(l x), если l/2 < x < l. 1, если < x 1, 2 x, если 1 x 2. Ответы 14. f(x) = 4 cos(2n 1)πx. π 2 (2n 1) f(x) = sh sh4 (1) n nπx cos 16 + π 2 n f(x) = cos 2nπx. π 2 n f(x) = 2 π + 8 π (1) n n 1 4n 2 cosnπx. 39

40 18. f(x) = 8 (1) n n sen nπx. π 1 4n (1) n 2n + 1 cos πx. π 2n πn 2πnx π 2 sen2 cos n π sen ωx 2 cos 2nωx π 4n 2 1. (l 22. a) f(x) = al 4 2) 1 (4n 2)πx cos, π 2 (2n 1) 2 l b) f(x) = 4al (1) n 1 (2n 1) πx sen. π 2 (2n 1) 2 l 23. a) f(x) = (cos π π 2 2 x 2 2 cos 2π 2 2 x cos 3π 2 2 x cos 5π), 2 2 x... b) f( x) = 4 (sen π π 2 2 x 1 3 sen 3π)+ 2 2 x (sen π π 2 x cos 2π) 2 x Forma complexa da série de Fourier Decomposição f(x) = c n e inx, onde c n = 1 2π f (x)e inx dx, n = ±1, ±2,..., é chamada de forma complexa da série de Fourier. A função se expande em uma série de Fourier complexa sob as mesmas condições sob as quais ela se expande em uma série de Fourier real. quatro

41 EXEMPLO 1. Encontre a série de Fourier na forma complexa da função dada pela fórmula f(x) = e ax no intervalo [, π), onde a é um número real. Solução. Vamos calcular os coeficientes: = c n = 1 2π f(x)e inx dx = 1 2π e (a in)x dx = 1 ((1) n e aπ (1) n e aπ) = (1)n sh aπ. 2π(a in) π(a in) A série complexa de Fourier da função f tem a forma f(x) sh aπ π n= (1) n a in einx. Vamos verificar que a função f(x) é suave por partes: no intervalo (, π) ela é continuamente diferenciável, e nos pontos x = ±π existem limites finitos (5), (6) lim h + ea( +h) = e aπ, lim h + ea(π h) = e aπ, e a(+h) e a(+) lim h + h = ae aπ e a(π h) e a(π), lim h + h = ae aπ. Portanto, a função f(x) pode ser representada por uma série de Fourier sh aπ π n= (1) n a em einx, que converge para a soma: ( e S(x) = ax se π< x < π, ch a, если x = ±π. 41

42 EXEMPLO 11. Encontre a série de Fourier na forma complexa e real da função dada pela fórmula f(x) = 1 a 2 1 2a cosx + a2, onde a< 1, a R. Решение. Функция f(x) является четной, поэтому для всех n b n =, а a n = 2 π f(x) cosnxdx = 2 (1 a2) π cos nxdx 1 2a cosx + a 2. Не будем вычислять такой сложный интеграл, а применим следующий прием: 1. используя формулы Эйлера sin x = eix e ix 2i = z z 1, cosx = eix + e ix 2i 2 = z + z 1, 2 где z = e ix, преобразуем f(x) к рациональной функции комплексной переменной z; 2. полученную рациональную функцию разложим на простейшие дроби; 3. разложим простейшую дробь по формуле геометрической прогрессии; 4. упростим полученную формулу. Итак, по формулам Эйлера получаем = f(x) = 1 a 2 1 a(z + z 1) + a 2 = (a 2 1)z (z a)(z a 1) = a z a az. (14) 42

43 Lembre-se que a soma de uma progressão geométrica infinita com denominador q (q< 1) вычисляется по формуле: + n= q n = 1 1 q. Эта формула верна как для вещественных, так и для комплексных чисел. Поскольку az = a < 1 и a/z = a < 1, то az = + a n z n = a n e inx, a z a = a z 1 1 a/z = a z n= + n= a n z = + n n= n= a n+1 z = + a n+1 e i(n+1)x. n+1 После замены переменной (n + 1) = k, < k < 1, получим: 1 a z a = a k e ikx. Следовательно, f(x) + n= k= c n e inx, где c n = n= { a n, если n, a n, если n <, то есть c n = a n. Поскольку функция f(x) непрерывна, то в силу теоремы о поточечной сходимости имеет место равенство: f(x) = + n= a n e inx. Тем самым мы разложили функцию f(x) в ряд Фурье в комплексной форме. 43

44 Agora vamos encontrar a série de Fourier na forma real. Para fazer isso, agrupamos os termos com números n e n para n: a n e inx + a n e inx = 2a neinx + e inx Como c = 1, então 2 = 2a n cos nx. f(x) = 1 a 2 1 2a cosx + a = a n cosnx. 2 Esta é uma série de Fourier na forma real da função f(x). Assim, sem calcular uma única integral, encontramos a série de Fourier da função. Ao fazer isso, calculamos uma integral rígida dependendo do parâmetro cos nxdx 1 2a cosx + a = 2 π an 2 1 a2, a< 1. (15) ПРИМЕР 12. Найдем ряд Фурье в комплексной и вещественной форме функции, заданной формулой a sin x f(x) = 1 2a cosx + a2, a < 1, a R. Решение. Функция f(x) является нечетной, поэтому для всех n a n = и b n = 2 π f(x) sin nxdx = 2a π sin x sin nxdx 1 2a cosx + a 2. Чтобы записать ряд Фурье нужно вычислить сложные интегралы или воспользоваться приемом, описанным выше. Поступим вторым способом: 44

45 a(z z 1) f(x) = 2i (1 a(z z 1) + a 2) = i 2 + i (a + a 1)z 2 2 (z a)(z a 1) = = i 2 + i () a 2 z a + a 1. z a 1 Expandimos cada uma das frações simples de acordo com a fórmula de progressão geométrica: + a z a = a 1 z 1 a = a a n z z n, n= z a 1 z a = az = a n z n. n= Isso é possível porque az = a/z = a< 1. Значит + ia n /2, если n <, f(x) c n e inx, где c n =, если n =, n= ia n /2, если n >, ou, mais brevemente, c n = 1 2i a n sgnn. Assim, a série de Fourier na forma complexa é encontrada. Agrupando os termos com os números n e n, obtemos a série de Fourier da função na forma real: = f(x) = + a sin x 1 2a cosx + a + 2 (1 2i an e inx 1 2i an e inx n= +) = c n e inx = a n sin nx. Novamente, conseguimos calcular a seguinte integral complexa: sen x sen nxdx 1 2a cosx + a 2 = π an 1. (16) 45

46 PROBLEMA 24. Usando (15), calcule a integral cos nxdx 1 2a cosx + a 2 para real a, a > Usando (16), calcule a integral sen x sen nxdx para real a, a > a cosx + a2 Em problemas , encontre a série Fourier na forma complexa para funções. 26. f(x) = sgn x, π< x < π. 27. f(x) = ln(1 2a cosx + a 2), a < 1. 1 a cosx 28. f(x) = 1 2a cosx + a2, a < Докажите, что функция f, определенная в промежутке [, π], вещественнозначна, если и только если коэффициенты c n ее комплексного ряда Фурье связаны соотношениями c n = c n, n =, ±1, ±2, Докажите, что функция f, определенная в промежутке [, π], является четной (т. е. удовлетворяет соотношению f(x) = f(x)), если и только если коэффициенты c n ее комплексного ряда Фурье связаны соотношениями c n = c n, n = ±1, ±2, Докажите, что функция f, определенная в промежутке [, π], является нечетной (т. е. удовлетворяет соотношению f(x) = f(x)), если и только если коэффициенты c n ее комплексного ряда Фурье связаны соотношениями c n = c n, n =, ±1, ±2,.... Ответы 1 2π 24. a n a π a n i + e 2inx, где подразумевается, что слагаемое, соответствующее n =, пропущено. π n n= a n n cosnx. 28. a n cosnx. n= 46

47 5. Teorema da igualdade de Lyapunov (igualdade de Lyapunov). Seja uma função f: [, π] R tal que f 2 (x) dx< +, и пусть a n, b n ее коэффициенты Фурье. Тогда справедливо равенство, a (a 2 n + b2 n) = 1 π называемое равенством Ляпунова. f 2 (x) dx, ПРИМЕР 13. Напишем равенство Ляпунова для функции { 1, если x < a, f(x) =, если a < x < π и найдем с его помощью суммы числовых рядов + sin 2 na n 2 и + Решение. Очевидно, 1 (2n 1) 2. 1 π f 2 (x) dx = 1 π a a dx = 2a π. Так как f(x) четная функция, то для всех n имеем b n =, a = 2 π f(x) dx = 2 π a dx = 2a π, 47

48 a n = 2 π f(x) cosnxdx = 2 π a cos nxdx = 2 sen na πn. Portanto, a igualdade de Lyapunov para a função f(x) assume a forma: 2 a 2 π + 4 sen 2 na = 2a 2 π 2 n 2 π. Da última igualdade para a π encontramos sen 2 na n 2 = a(π a) 2 Assumindo a = π 2, obtemos sen2 na = 1 para n = 2k 1 e sen 2 na = para n = 2k. Portanto, k=1 1 (2k 1) 2 = π2 8. EXEMPLO 14. Vamos escrever a igualdade de Lyapunov para a função f(x) = x cosx, x [, π], e usá-la para encontrar a soma do número série (4n 2 + 1) 2 (4n 2 1) 4. 1 π Solução. Cálculos diretos dão = π π f 2 (x) dx = 1 π x 2 cos 2 xdx = 1 π x sen 2xdx = π π x cos x = π x 21 + cos 2x dx = 2 π 1 4π cos 2xdx =

49 Como f(x) é uma função par, então para todo n temos b n =, a n = 2 π = 1 π 1 = π(n + 1) = f(x) cosnxdx = 2 π 1 cos(n + 1) )x π (n + 1) 2 x cosxcosnxdx = x (cos(n + 1)x + cos(n 1)x) dx = 1 π sen(n + 1)xdx sen(n 1)xdx = π(n 1) π π 1 + cos(n 1)x = π(n 1) 2 1 (= (1) (n+1) 1) 1 (+ (1) (n+1) 1) = π(n + 1) 2 π(n 1) 2 () = (1)(n+1) 1 1 π (n + 1) + 1 = 2 (n 1) 2 = 2 (1)(n+1) 1 n k π (n 2 1) = π (4k 2 1) 2 se n = 2k, 2 se n = 2k + 1. O coeficiente a 1 deve ser calculado separadamente, pois na fórmula geral para n = 1 o denominador da fração se anula . = 1 π a 1 = 2 π f(x) cosxdx = 2 π x(1 + cos 2x)dx = π 2 1 2π 49 x cos 2 xdx = sen 2xdx = π 2.

50 Assim, a igualdade de Lyapunov para a função f(x) tem a forma: 8 π + π (4n 2 + 1) 2 π 2 (4n 2 1) = π 2 1) = π π PROBLEMA 32. Escreva a igualdade de Lyapunov para a função ( x f(x) = 2 πx se x< π, x 2 πx, если π < x. 33. Напишите равенства Ляпунова для функций f(x) = cos ax и g(x) = sin ax, x [, π]. 34. Используя результат предыдущей задачи и предполагая, что a не является целым числом, выведите следующие классические разложения функций πctgaπ и (π/ sin aπ) 2 по рациональным функциям: πctgaπ = 1 a + + 2a a 2 n 2, (π) = sin aπ (a n) 2. n= 35. Выведите комплексную форму обобщенного равенства Ляпунова. 36. Покажите, что forma complexa A equação de Lyapunov é válida não apenas para funções de valor real, mas também para funções de valor complexo. 5

51 π (2n + 1) = π sen 2απ 2απ = 2sen2 απ α 2 π 2 Respostas + 4 sen2 απ π 2 α 2 (α 2 n 2) 2; sin 2απ 1 2απ = απ n 2 4sin2 π 2 (α 2 n 2) 2. 1 π 35. f(x)g(x) dx= c n d n, onde c n é o coeficiente de Fourier 2π de f(x), e d n é a função coeficiente de Fourier g(x). 6. Diferenciação de séries de Fourier Seja f: R R uma função 2π-periódica continuamente diferenciável. Sua série de Fourier tem a forma: f(x) = a 2 + (a n cos nx + b n sin nx). A derivada f (x) desta função será uma função contínua e 2π-periódica, para a qual uma série formal de Fourier pode ser escrita: f (x) a 2 + (a n cos nx + b n sin nx), onde a, a n , b n, n = 1 , 2,... Coeficientes de Fourier da função f (x). 51

52 Teorema (sobre a diferenciação termo a termo de séries de Fourier). Sob as suposições acima, são verdadeiras as igualdades a =, a n = nb n, b n = na n, n 1. EXEMPLO 15. Seja uma função suave por partes f(x) contínua no intervalo [, π]. Vamos provar que quando a condição f(x)dx = é satisfeita, a desigualdade 2 dx 2 dx, chamada desigualdade de Steklov, vale, e verificamos que a igualdade nela é realizada apenas para funções da forma f(x) = A cosx. Em outras palavras, a desigualdade de Steklov fornece condições sob as quais a pequenez da derivada (em rms) implica a pequenez da função (em rms). Solução. Vamos estender a função f(x) para o intervalo [, ] uniformemente. Denote a função estendida pelo mesmo símbolo f(x). Então a função continuada será contínua e suave por partes no intervalo [, π]. Como a função f(x) é contínua, então f 2 (x) é contínua no intervalo e 2 dx< +, следовательно, можно применить теорему Ляпунова, согласно которой имеет место равенство 1 π 2 dx = a () a 2 n + b 2 n. 52

53 Como a função continuada é par, então b n =, a = por condição. Conseqüentemente, a igualdade de Lyapunov assume a forma 1 π 2 dx = a 2 π n. (17) Certifique-se de que f (x) satisfaz a conclusão do teorema sobre a diferenciação termo a termo da série de Fourier, ou seja, que a =, a n = nb n, b n = na n, n 1. Deixe a derivada f (x) sofrer quebras nos pontos x 1, x 2,..., x N no intervalo [, π]. Denote x =, x N+1 = π. Vamos dividir o intervalo de integração [, π] em N +1 intervalos (x, x 1),..., (x N, x N+1), em cada um dos quais f(x) é continuamente diferenciável. Então, usando a propriedade de aditividade da integral e integrando por partes, temos: b n = 1 π = 1 π = 1 π f (x) sen nxdx = 1 π N f(x) sen nx j= N f(x) ) sen nx j= x j+1 x j x j+1 x j n n π N j= x j+1 x j x j+1 x j f (x) sen nxdx = f(x) cosnxdx = f(x) cosnxdx = = 1 π [( f(x 1) sen nx 1 f(x) sen nx) + + (f(x 2) sennx 2 f(x 1) sen nx 1)

54 + (f(x N+1) sen nx N+1 f(x N) sen nx N)] na n = = 1 π na n = = 1 π na n = na n. x j+1 a = 1 f (x) dx = 1 N f (x) dx = π π j= x j = 1 N x j+1 f(x) π = 1 (f(π) f()) = . x j π j= Da mesma forma, obtemos a n = nb n. Mostramos que o teorema da diferenciação termo a termo de séries de Fourier para uma função 2π periódica contínua por partes cuja derivada no intervalo [, π] sofre descontinuidades do primeiro tipo é verdadeiro. Então f (x) a 2 + (a n cosnx + b n sen nx) = (na n) sen nx, visto que a =, a n = nb n =, b n = na n, n = 1, 2,.... Porque 2dx< +, то по равенству Ляпунова 1 π 2 dx = 54 n 2 a 2 n. (18)

55 Como cada termo da série em (18) é maior ou igual ao termo correspondente da série em (17), então 2 dx 2 dx. Lembrando que f(x) é uma continuação par da função original, temos 2 dx 2 dx. O que prova a igualdade de Steklov. Agora vamos examinar para quais funções a igualdade é válida na desigualdade de Steklov. Se para pelo menos um n 2, o coeficiente a n é diferente de zero, então a 2 n< na 2 n. Следовательно, равенство a 2 n = n 2 a 2 n возможно только если a n = для n 2. При этом a 1 = A может быть произвольным. Значит в неравенстве Стеклова равенство достигается только на функциях вида f(x) = A cosx. Отметим, что условие πa = f(x)dx = (19) существенно для выполнения неравенства Стеклова, ведь если условие (19) нарушено, то неравенство примет вид: a a 2 n n 2 a 2 n, а это не может быть верно при произвольном a. 55

56 PROBLEMAS 37. Seja uma função suave por partes f(x) contínua no intervalo [, π]. Prove que sob a condição f() = f(π) = a desigualdade 2 dx 2 dx, também chamada de desigualdade de Steklov, vale, e certifique-se de que a igualdade nela vale apenas para funções da forma f(x) = B sen x . 38. Seja uma função f contínua no intervalo [, π] e tenha nele (com a possível exceção de apenas um número finito de pontos) uma derivada quadrada integrável f(x). Prove que se as condições f() = f(π) e f(x) dx = são satisfeitas, então a desigualdade 2 dx 2 dx, chamada de desigualdade de Wirtinger, é válida, e a igualdade nela ocorre apenas para funções do forma f(x) = A cosx + B senx. 56

57 7. Aplicação da série de Fourier para resolver equações diferenciais parciais Ao estudar um objeto real (fenômenos naturais, processo de produção, sistema de controle, etc.), dois fatores se tornam significativos: o nível de conhecimento acumulado sobre o objeto em estudo e o grau de desenvolvimento do aparato matemático. No estágio atual pesquisa científica, a seguinte cadeia foi desenvolvida: um fenômeno um modelo físico um modelo matemático. A formulação física (modelo) do problema é a seguinte: são identificadas as condições para o desenvolvimento do processo e os principais fatores que o influenciam. A formulação matemática (modelo) consiste em descrever os fatores e condições escolhidos na formulação física na forma de um sistema de equações (algébricas, diferenciais, integrais, etc.). Um problema é dito bem colocado se, em um determinado espaço funcional, a solução do problema existe, depende única e continuamente das condições iniciais e de contorno. O modelo matemático não é idêntico ao objeto em consideração, mas é sua descrição aproximada Derivação da equação das pequenas vibrações transversais livres da corda Seguiremos o livro didático. Deixe as extremidades da corda serem fixas e a própria corda esticada. Se a corda for retirada do equilíbrio (por exemplo, puxando ou batendo), a corda começará 57

58 hesitar. Vamos supor que todos os pontos da corda se movem perpendicularmente à sua posição de equilíbrio (vibrações transversais), e a cada momento a corda está no mesmo plano. Tomemos neste plano o sistema coordenadas retangulares xou. Então, se no instante inicial t = o fio estava localizado ao longo do eixo Ox, então u significará o desvio do fio da posição de equilíbrio, ou seja, a posição do ponto do fio com a abcissa x em um tempo arbitrário t corresponde ao valor da função u(x, t). Para cada valor fixo de t, o gráfico da função u(x, t) representa a forma da corda vibrante no instante t (Fig. 32). A um valor constante de x, a função u(x, t) fornece a lei do movimento de um ponto com a abcissa x ao longo de uma linha reta paralela ao eixo Ou, a derivada u t é a velocidade desse movimento, e a segunda derivada 2 u t 2 é a aceleração. Arroz. 32. Forças aplicadas a uma seção infinitamente pequena de uma corda Vamos escrever uma equação que a função u(x, t) deve satisfazer. Para fazer isso, fazemos algumas suposições mais simplificadoras. Vamos supor que a string é absolutamente flexível.

59 coy, ou seja, vamos supor que a corda não resiste à flexão; isso significa que as tensões que surgem na corda são sempre direcionadas tangencialmente ao seu perfil instantâneo. A corda é considerada elástica e sujeita à lei de Hooke; isso significa que a mudança na magnitude da força de tração é proporcional à mudança no comprimento da corda. Suponhamos que a corda seja homogênea; isso significa que sua densidade linear ρ é constante. Desprezamos as forças externas. Isso significa que estamos considerando oscilações livres. Estudaremos apenas pequenas vibrações de uma corda. Se denotarmos por ϕ(x, t) o ângulo entre o eixo das abcissas e a tangente à corda no ponto com a abcissa x no instante t, então a condição para pequenas oscilações é que o valor de ϕ 2 (x, t) pode ser desprezado em comparação com ϕ (x, t), ou seja, ϕ 2. Como o ângulo ϕ é pequeno, então cos ϕ 1, ϕ sen ϕ tg ϕ u, portanto, o valor (u x x,) 2 também pode ser negligenciado. Segue-se imediatamente disso que, no processo de oscilação, podemos desprezar a mudança no comprimento de qualquer seção da corda. De fato, o comprimento de um pedaço de corda M 1 M 2 projetado no intervalo do eixo x, onde x 2 = x 1 + x, é igual a l = x 2 x () 2 u dx x. x Vamos mostrar que, sob nossas suposições, o valor da força de tração T será constante ao longo de todo o fio. Para fazer isso, pegamos uma parte da corda M 1 M 2 (Fig. 32) no tempo t e substituímos a ação das partes descartadas

60 kov pelas forças de tração T 1 e T 2. Como, de acordo com a condição, todos os pontos da corda se movem paralelamente ao eixo Ou e não há forças externas, a soma das projeções das forças de tração no eixo Ox deve ser igual a zero: T 1 cosϕ(x 1, t) + T 2 cosϕ(x 2, t) =. Assim, devido à pequenez dos ângulos ϕ 1 = ϕ(x 1, t) e ϕ 2 = ϕ(x 2, t), concluímos que T 1 = T 2. Denote Significado geral T 1 \u003d T 2 a T. Agora calculamos a soma das projeções F u das mesmas forças no eixo Ou: F u \u003d T sin ϕ (x 2, t) T sin ϕ (x 1, t) . (2) Como para ângulos pequenos sen ϕ(x, t) tg ϕ(x, t), e tg ϕ(x, t) u(x, t)/x, a equação (2) pode ser reescrita como F u T (tan ϕ(x 2, t) tan ϕ(x 1, t)) (u T x (x 2, t) u) x (x 1, t) x x T 2 u x 2(x 1, t) x . Como o ponto x 1 é escolhido arbitrariamente, então F u T 2 u x2(x, t) x. Depois que todas as forças que atuam na seção M 1 M 2 são encontradas, aplicamos a segunda lei de Newton, segundo a qual o produto da massa pela aceleração é igual à soma de todas as forças atuantes. A massa de um pedaço de corda M 1 M 2 é igual a m = ρ l ρ x, e a aceleração é igual a 2 u(x, t). A equação t 2 de Newton assume a forma: 2 u t (x, t) x = u 2 α2 2 x2(x, t) x, onde α 2 = T ρ é um número positivo constante. 6

61 Reduzindo por x, obtemos 2 u t (x, t) = u 2 α2 2 x2(x, t). (21) Como resultado, obtivemos uma equação diferencial parcial homogênea linear de segunda ordem com coeficientes constantes. É chamada de equação de vibração de cordas ou equação de onda unidimensional. A Equação (21) é essencialmente uma reformulação da lei de Newton e descreve o movimento de uma corda. Mas na formulação física do problema, havia requisitos de que as extremidades da corda fossem fixas e a posição da corda em algum ponto no tempo fosse conhecida. Vamos escrever essas condições em equações da seguinte forma: a) vamos supor que as extremidades da corda estão fixas nos pontos x = e x = l, ou seja, vamos supor que para todo t as relações u(, t) = , u(l, t) = ; (22) b) assumiremos que no instante t = a posição da corda coincide com o gráfico da função f(x), ou seja, assumiremos que para todo x [, l] a igualdade u(x, ) = f(x); (23) c) assumiremos que no instante t = o ponto do fio com a abcissa x recebe velocidade g(x), ou seja, assumiremos que u(x,) = g(x). (24) As relações (22) são chamadas de condições de contorno e as relações (23) e (24) são chamadas de condições iniciais. Modelo matemático da pequena transversal livre 61

62 vibrações de cordas é que é necessário resolver a equação (21) com condições de contorno (22) e condições iniciais (23) e (24) Solução da equação de pequenas vibrações transversais livres da corda pelo método de Fourier< t <, удовлетворяющие граничным условиям (22) и начальным условиям (23) и (24), будем искать методом Фурье (называемым также методом разделения переменных). Метод Фурье состоит в том, что частные решения ищутся в виде произведения двух функций, одна из которых зависит только от x, а другая только от t. То есть мы ищем решения уравнения (21), которые имеют специальный вид: u(x, t) = X(x)T(t), (25) где X дважды непрерывно дифференцируемая функция от x на [, l], а T дважды непрерывно дифференцируемая функция от t, t >. Substituindo (25) em (21), obtemos: X T = α 2 X T, (26) ou T (t) α 2 T(t) = X (x) X(x). (27) Diz-se que houve uma separação de variáveis. Como x e t não dependem um do outro, o lado esquerdo em (27) não depende de x, mas o lado direito não depende de t, e o valor total dessas razões é 62

63 deve ser constante, o que denotamos por λ: T (t) α 2 T(t) = X (x) X(x) = λ. A partir disso, obtemos dois comuns equações diferenciais: X (x) λx(x) =, (28) T (t) α 2 λt(t) =. (29) Neste caso, as condições de contorno (22) assumem a forma X()T(t) = e X(l)T(t) =. Como eles devem ser preenchidos para todo t, t >, então X() = X(l) =. (3) Vamos encontrar soluções para a equação (28) satisfazendo as condições de contorno (3). Vamos considerar três casos. Caso 1: λ >. Denote λ = β 2. A equação (28) assume a forma X (x) β 2 X(x) =. Sua equação característica k 2 β 2 = tem raízes k = ±β. Consequentemente, decisão comum a equação (28) tem a forma X(x) = C e βx + De βx. Devemos escolher as constantes C e D para que as condições de contorno (3) sejam atendidas, ou seja, X() = C + D =, X(l) = C e βl + De βl =. Desde β, então este sistema de equações tem única decisão C=D=. Portanto, X(x) e 63

64 u(x, t). Assim, no caso 1 obtivemos uma solução trivial, que não consideraremos mais adiante. Caso 2: λ =. Então a equação (28) assume a forma X(x) = e sua solução é obviamente dada pela fórmula: X(x) = C x+d. Substituindo esta solução nas condições de contorno (3), obtemos X() = D = e X(l) = Cl =, portanto C = D =. Portanto, X(x) e u(x, t), e novamente temos uma solução trivial. Caso 3: λ<. Обозначим λ = β 2. Уравнение (28) принимает вид: X (x)+β 2 X(x) =. Его характеристическое уравнение имеет вид k 2 + β 2 =, а k = ±βi являются его корнями. Следовательно, общее решение уравнения (28) в этом случае имеет вид X(x) = C sin βx + D cosβx. В силу граничных условий (3) имеем X() = D =, X(l) = C sin βl =. Поскольку мы ищем нетривиальные решения (т. е. такие, когда C и D не равны нулю одновременно), то из последнего равенства находим sin βl =, т. е. βl = nπ, n = ±1, ±2,..., n не равно нулю, так как сейчас мы рассматриваем случай 3, в котором β. Итак, если β = nπ (nπ) 2, l, т. е. λ = то существуют l решения X n (x) = C n sin πnx, (31) l C n произвольные постоянные, уравнения (28), не равные тождественно нулю. 64

65 A seguir, atribuiremos a n apenas valores positivos n = 1, 2,..., pois para n negativo serão obtidas soluções da mesma forma (nπ). Os valores λ n = são chamados autovalores, e as funções X n (x) = C n sin πnx autofunções da equação diferencial (28) com condições de contorno (3). Agora vamos resolver a equação (29). Para ele, a equação característica tem a forma k 2 α 2 λ =. (32) l 2 Como descobrimos acima que soluções não triviais X(x) da Eq. (28) existem apenas para λ negativo igual a λ = n2 π 2, são esses λ que consideraremos abaixo. As raízes da equação (32) são k = ±iα λ, e as soluções da equação (29) têm a forma: T n (t) = A n sin πnαt + B n cos πnαt, (33) l l onde A n e B n são constantes arbitrárias. Substituindo as fórmulas (31) e (33) em (25), encontramos soluções particulares da equação (21) que satisfazem as condições de contorno (22): (u n (x, t) = B n cos πnαt + A n sin πnαt) C n sen pnx. l l l Inserindo o fator C n entre parênteses e introduzindo a notação C n A n = b n e B n C n = a n, escrevemos u n (X, T) como (u n (x, t) = a n cos πnαt + b n sin πnαt ) sin pnx. (34) l l l 65

66 As vibrações da corda correspondentes às soluções u n (x, t) são chamadas de vibrações naturais da corda. Como a equação (21) e as condições de contorno (22) são lineares e homogêneas, então uma combinação linear de soluções (34) (u(x, t) = a n cos πnαt + b n sen πnαt) sen πnx (35) l l l será um solução da equação (21 ) satisfazendo as condições de contorno (22) com uma escolha especial dos coeficientes a n e b n, o que garante a convergência uniforme da série. Agora escolhemos os coeficientes a n e b n da solução (35) para que ela satisfaça não apenas as condições de contorno, mas também as condições iniciais (23) e (24), onde f(x), g(x) são dadas funções ( além disso, f() = f (l) = g() = g(l) =). Assumimos que as funções f(x) eg(x) satisfazem as condições de expansão de Fourier. Substituindo o valor t = em (35), obtemos u(x,) = a n sen πnx l = f(x). Diferenciando a série (35) em relação a t e substituindo t =, obtemos u t (x,) = πnα b n sen πnx l l = g(x), e esta é a expansão das funções f(x) e g(x) em série de Fourier. Portanto, a n = 2 l l f(x) sen πnx l dx, b n = 2 l g(x) sen πnx dx. πnα l (36) 66

67 Substituindo as expressões dos coeficientes a n e b n na série (35), obtemos uma solução para a equação (21) que satisfaz as condições de contorno (22) e as condições iniciais (23) e (24). Assim, resolvemos o problema das pequenas vibrações transversais livres de uma corda. Esclareçamos o significado físico das autofunções u n (x, t) do problema das vibrações livres de uma corda, definidas pela fórmula (34). Vamos reescrevê-lo como onde u n (x, t) = α n cos πnα l α n = a 2 n + b2 n, (t + δ n) sen πnx, (37) l πnα δ n = arctg b n. l a n A fórmula (37) mostra que todos os pontos da corda realizam oscilações harmônicas com a mesma frequência ω n = πnα e fase πnα δ n. A amplitude de oscilação depende de l l a abcissa x da ponta da corda e é igual a α n sen πnx. Com essa oscilação, todos os pontos da corda atingem simultaneamente seu desvio máximo l em uma direção ou outra e passam simultaneamente na posição de equilíbrio. Tais oscilações são chamadas de ondas estacionárias. Uma onda estacionária terá n + 1 pontos fixos dados pelas raízes da equação sen πnx = no intervalo [, l]. Os pontos fixos são chamados de nós da onda estacionária. No meio entre os nós - l mi são os pontos em que os desvios atingem um máximo; tais pontos são chamados antinós. Cada corda pode ter suas próprias oscilações de frequências estritamente definidas ω n = πnα, n = 1, 2,.... Essas frequências são chamadas de frequências naturais da corda. O tom l mais baixo que uma corda pode produzir é determinado por ela mesma.

68 baixa frequência natural ω 1 = π T e é chamado de tom fundamental da corda. Os tons restantes correspondentes às frequências l ρ ω n, n = 2, 3,..., são chamados de harmônicos ou harmônicos. Para maior clareza, descreveremos os perfis típicos de uma corda que emite o tom fundamental (Fig. 33), o primeiro harmônico (Fig. 34) e o segundo harmônico (Fig. 35). Arroz. Fig. 33. Perfil da corda que emite o tom fundamental. Fig. 34. Perfil de uma corda emitindo o primeiro harmônico. Fig. 35. Perfil de uma corda emitindo um segundo harmônico Se a corda realiza vibrações livres determinadas pelas condições iniciais, então a função u(x, t) é representada, como pode ser visto na fórmula (35), como uma soma de harmônicos individuais. Assim oscilação arbitrária 68

A 69ª corda é uma superposição de ondas estacionárias. Neste caso, a natureza do som da corda (tom, intensidade do som, timbre) dependerá da razão entre as amplitudes dos harmônicos individuais. ouvido como um som emitido por uma corda. A força do som é caracterizada pela energia ou amplitude das vibrações: quanto maior a energia, maior a força do som. O tom de um som é determinado por sua frequência ou período de oscilação: quanto mais alta a frequência, mais alto o som. O timbre do som é determinado pela presença de harmônicos, a distribuição de energia sobre os harmônicos, ou seja, o método de excitação das vibrações. As amplitudes dos harmônicos são, em geral, menores que a amplitude da fundamental, e as fases dos harmônicos podem ser arbitrárias. Nosso ouvido não é sensível à fase de oscilações. Compare, por exemplo, as duas curvas da Fig. 36, emprestado de . Trata-se de uma gravação de som com o mesmo tom fundamental, extraído do clarinete (a) e do piano (b). Ambos os sons não são simples oscilações senoidais. A frequência fundamental do som em ambos os casos é a mesma e isso cria o mesmo tom. Mas os padrões de curva são diferentes porque diferentes harmônicos são sobrepostos ao tom fundamental. De certa forma, esses desenhos mostram o que é o timbre. 69


MINISTÉRIO DA EDUCAÇÃO E CIÊNCIA DA RÚSSIA Orçamento do Estado Federal Instituição Educacional de Ensino Superior Profissional MATI Universidade Tecnológica Estatal Russa em homenagem a K. E. Tsiolkovsky

Agência Federal de Educação Instituição Educacional Estadual Federal de Educação Profissional Superior UNIVERSIDADE FEDERAL DO SUL R. M. Gavrilova, G. S. Kostetskaya Methodical

Ministério da Educação da República da Bielorrússia Vitebsk State Technological University Tópico. "Linhas" Departamento de Matemática Teórica e Aplicada. desenvolvido pela Assoc. E. B. Dunina. Principal

Aula 4. Análise harmônica. Série de Fourier Funções periódicas. Análise harmônica Em ciência e tecnologia, muitas vezes temos que lidar com fenômenos periódicos, ou seja, aqueles que se repetem

UNIVERSIDADE TÉCNICA DE AVIAÇÃO CIVIL DO ESTADO DE MOSCOU V.M. Lyubimov, E. A. Zhukova, V. A. Ukhova, Yu.A. Shurinov

CONTEÚDO Série de Fourier 4 O conceito de uma função periódica 4 Polinômio trigonométrico 6 3 Sistemas ortogonais de funções 4 Série de Fourier trigonométrica 3 5 Série de Fourier para funções pares e ímpares 6 6 Decomposição

INTEGRAL DEFINIDA. Somas Integrais e Integral Definida Seja uma função y = f () definida no segmento [, b ], onde< b. Разобьём отрезок [, b ] с помощью точек деления на n элементарных

TEORIA DAS SÉRIES A teoria das séries é o componente mais importante da análise matemática e encontra aplicações teóricas e inúmeras aplicações práticas. Distinguir entre séries numéricas e funcionais.

TÓPICO V SÉRIE DE FOURIER AULA 6 Expansão de uma função periódica em uma série de Fourier Muitos processos que ocorrem na natureza e na tecnologia têm a propriedade de se repetir em certos intervalos Tais processos

6 Série de Fourier 6 Sistemas ortogonais de funções Série de Fourier em termos de um sistema ortogonal de funções As funções ϕ () e ψ (), definidas e integráveis ​​no segmento [, ], são ditas ortogonais neste segmento se

Agência Federal de Transporte Ferroviário Universidade Estadual do Ural do Departamento de Transporte Ferroviário "Matemática Superior e Aplicada" N. P. Chuev Elementos de Análise Harmônica Metódica

UNIVERSIDADE DO ESTADO DA BIELORRÚSSIA FACULDADE DE MATEMÁTICA APLICADA E CIÊNCIAS DA INFORMAÇÃO Departamento de Matemática Superior Auxiliar de ensino para alunos da Faculdade de Matemática Aplicada e Informática

Explicações para o texto: o sinal é lido como "equivalente" e significa que as equações à direita do sinal e à esquerda do sinal têm o mesmo conjunto de soluções, o sinal IR denota o conjunto dos números reais, o sinal DENTRO

EQUAÇÕES DA FÍSICA MATEMÁTICA 1. Equações diferenciais parciais

1 2 Índice 1 Série de Fourier 5 1.1 Série trigonométrica de Fourier .......... 5 1.2 Apenas sin & cos ............. ............ 7 1.3 Série de Fourier na forma complexa .......... 11 1.4 f(x) = c k?......... ......

82 4. Seção 4. Séries funcionais e de potência 4.2. Lição 3 4.2. Lição 3 4.2.. Expansão de Taylor de uma função DEFINIÇÃO 4.2.. Seja a função y = f(x) infinitamente diferenciável em alguma vizinhança

Aula 8 4 Problema de Sturm-Liouville

MINISTÉRIO DA EDUCAÇÃO E CIÊNCIA DA RÚSSIA ESTADO FEDERAL INSTITUIÇÃO EDUCACIONAL DE ENSINO SUPERIOR PROFISSIONAL "UNIVERSIDADE TÉCNICA DO ESTADO DE SAMARA" Departamento de Matemática Aplicada

Integrabilidade de uma função (de acordo com Riemann) e uma integral definida Exemplos de resolução de problemas 1. A função constante f(x) = C é integrável em , pois para quaisquer partições e qualquer escolha de pontos ξ i a integral

INSTRUÇÕES METODOLÓGICAS PARA TAREFAS DE CÁLCULO NO CURSO DE MATEMÁTICA SUPERIOR "SÉRIE EQUAÇÕES DIFERENCIAIS ORDINÁRIAS SÉRIE INTEGRAL DUPLA" PARTE III SÉRIE TEMÁTICA Índice Série Série numérica Convergência e divergência

LINHAS. Linhas numéricas. Definições básicas Seja dada uma sequência infinita de números A expressão (soma infinita) a, a 2,..., a n,... a i = a + a 2 + + a n +... () i= é chamada de série numérica. Números

Título Introdução. Conceitos básicos.... 4 1. Equações integrais de Volterra... 5 Opções de trabalhos de casa.... 8 2. Resolvente da equação integral de Volterra. 10 opções de trabalhos de casa.... 11

Aula 3 Série de Taylor e Maclaurin Aplicação de séries de potências Expansão de funções em séries de potências Séries de Taylor e Maclaurin Para aplicações, é importante ser capaz de expandir uma determinada função em uma série de potências, essas funções

35 7 Série trigonométrica de Fourier Série de Fourier para funções periódicas com período T. Seja f(x) uma função periódica contínua por partes com período T. Considere o sistema trigonométrico básico

COMER. ORE ANÁLISE MATEMÁTICA. SÉRIE NUMÉRICA E FUNCIONAL NOVOSIBIRSK 200 2 MINISTÉRIO DA EDUCAÇÃO E CIÊNCIA DA RUSSA SEI HPE "UNIVERSIDADE PEDAGÓGICA DO ESTADO DE NOVOSIBIRSK" E.M. Rudoy ANÁLISE MATEMÁTICA.

Eu claro, tarefa. Prove que a função de Riemann, se 0, m m R(), se, m, m 0, e a fração é irredutível, 0, se irracional, é descontínua em todo ponto racional e contínua em todo irracional. Solução.

1. Eletrostática 1 1. Eletrostática Lição 6 Separação de variáveis ​​em coordenadas cartesianas 1.1. (Problema 1.49) O plano z é carregado com densidade σ (x, y) = σ sen (αx) sen (βy), onde σ, α, β são constantes.

Ch Série de potências a a a Uma série de potências da forma a a a a a () é chamada de série de potências, onde, a, são constantes, chamadas de coeficientes da série. Às vezes, uma série de potências de uma forma mais geral é considerada: a a (a) a ( a) a (a) (), onde

S A Lavrenchenko wwwwrkoru Palestra Transformada de Fourier Conceito de transformação integral O método de transformações integrais é um dos métodos poderosos da física matemática e é uma solução poderosa

Cálculo diferencial Introdução à análise matemática Sequência e limite de função. Divulgação de incertezas internas. Função derivada. Regras de diferenciação. Aplicação da derivada

PALESTRA N 7 .Poder

Faculdade de Metalurgia Departamento de Matemática Superior

9. Antiderivada e integral indefinida 9.. Seja a função f() dada no intervalo I R. A função F() é chamada de antiderivada f() no intervalo I, se F() = f() para qualquer I, e a antiderivada

Instituto de Física e Tecnologia de Moscou (Universidade Estadual) O.V. Besov TRIGONOMETRIC FOURIER SERIES Auxiliar de ensino Moscou, 004 Compilado por O.V.Besov UDC 517. Série trigonométrica

8. Série de potências 8.. Uma série funcional da forma c n (z) n, (8.) n= onde c n é uma sequência numérica, R é um número fixo e z R é chamado de série de potências com coeficientes c n . Mudando as variáveis

Departamento de Matemática e Informática Elementos de Matemática Superior Complexo educacional e metodológico para alunos do ensino médio profissionalizante que estudam usando tecnologias a distância Módulo Cálculo Diferencial Compilado por:

1. Integral definido 1.1. Seja f uma função limitada definida no segmento [, b] R. Uma partição do segmento [, b] é um conjunto de pontos τ = (x, x 1,..., x n 1, x n ) [, b ] tal que = x< x 1 < < x n 1

QUESTÕES E TAREFAS TÍPICAS para o exame final da disciplina "Análise Matemática" Matemática Aplicada Na prova oral, o aluno recebe duas questões teóricas e duas tarefas Total 66 questões por ano

Módulo Tópico Sequências e séries de funções Propriedades de convergência uniforme de sequências e séries Séries de potências Aula Definições de sequências e séries de funções Uniformemente

~ ~ Integrais indefinidas e definidas O conceito de antiderivada e integral indefinida. Definição: Uma função F é chamada de antiderivada em relação a uma função f se essas funções estiverem relacionadas da seguinte forma

Ministério da Educação e Ciência da Federação Russa Instituição Educacional Orçamentária do Estado Federal de Educação Profissional Superior "Universidade Industrial do Estado da Sibéria"

EQUAÇÕES QUADRÁTICAS

CENTRO EDUCACIONAL E CIENTÍFICO MILITAR DA FORÇA AÉREA "ACADEMIA DA FORÇA AÉREA em homenagem ao Professor N. E. ZHUKOVSKY e Yu. A. GAGARIN" EXEMPLOS

AGÊNCIA FEDERAL DE EDUCAÇÃO INSTITUIÇÃO ESTADUAL DE ENSINO SUPERIOR DE ENSINO PROFISSIONAL Universidade Estadual de Moscou de Engenharia de Instrumentos e Departamento de Informática do Superior

Capítulo 5. Série de Fourier 5 .. Lição 5 5 ... Definições básicas Uma série funcional da forma a 2 + (a k cos x + b k si x) (5 ..) é chamada de

Série de Fourier Sistemas ortogonais de funções Do ponto de vista da álgebra, a igualdade onde são funções de uma dada classe e são coeficientes de R ou C significa simplesmente que o vetor é uma combinação linear de vetores B

3724 SÉRIE DE INTEGRADOS MÚLTIPLOS E CURVILINEARES 1 PROGRAMA DE TRABALHO DAS SEÇÕES "SÉRIE DE INTEGRADOS MÚLTIPLOS E CURVILINEARES" 11 Série numérica O conceito de uma série numérica Propriedades da série numérica Um critério necessário para a convergência

DIFERENCIAÇÃO DE FUNÇÕES DE UMA VARIÁVEL O conceito de derivada, seu significado geométrico e físico Problemas que levam ao conceito de derivada Definição de Tangente S à reta y f (x) no ponto A x ; f(

EQUAÇÕES DIFERENCIAIS 1. Conceitos básicos Uma equação diferencial em relação a alguma função é uma equação que conecta esta função com suas variáveis ​​independentes e com suas derivadas.

EQUAÇÕES DIFERENCIAIS ORDINÁRIAS DE PRIMEIRA ORDEM Conceitos básicos Uma equação diferencial é uma equação na qual uma função desconhecida entra sob o sinal da derivada ou diferencial.

EQUAÇÕES DIFERENCIAIS Conceitos gerais As equações diferenciais têm inúmeras e muito diversas aplicações em mecânica, física, astronomia, tecnologia e em outros ramos da matemática superior (por exemplo,

Série funcional Série funcional sua soma e área do funcional o Seja uma sequência de funções k (k 1) dada na região Δ de números reais ou complexos

SISTEMAS DE POLINÔMICOS ORTOGONAL E SUAS APLICAÇÕES A. Polinômios de Chebyshev-Hermite

Aulas preparadas pelo Professor Adjunto Musina MV Definição Expressão da forma Séries numéricas e funcionais Séries numéricas: conceitos básicos (), onde se chama uma série numérica (ou apenas uma série) Números, membros de uma série (depende de

Ministério da Educação Geral e Profissional

Universidade Estadual de Turismo de Sochi

e negócios de resort

Instituto Pedagógico

Faculdade de Matemática

Departamento de Matemática Geral

TRABALHO DE GRADUAÇÃO

Série de Fourier e suas aplicações

em física matemática.

Completo por: Aluno do 5º ano

assinatura diurna

Especialidade 010100

"Matemáticas"

Kasperova N.S.

Cartão de estudante nº 95471

Orientador científico: professor associado, Ph.D.

assinatura técnica. Ciências

Pozin P.A.

Sóchi, 2000


1. Introdução.

2. O conceito de série de Fourier.

2.1. Determinação dos coeficientes da série de Fourier.

2.2. Integrais de funções periódicas.

3. Critérios de convergência das séries de Fourier.

3.1. Exemplos de expansão de funções em séries de Fourier.

4. Uma nota sobre a expansão de uma função periódica em uma série de Fourier

5. Série de Fourier para funções pares e ímpares.

6. Série de Fourier para funções com período 2 eu .

7. Expansão de Fourier de uma função não periódica.

Introdução.

Jean Baptiste Joseph Fourier - matemático francês, membro da Academia de Ciências de Paris (1817).

As primeiras obras de Fourier dizem respeito à álgebra. Já nas palestras de 1796, ele delineou um teorema sobre o número de raízes reais de uma equação algébrica situada entre limites dados (publ. 1820), em homenagem a ele; uma solução completa sobre o número de raízes reais de uma equação algébrica foi obtida em 1829 por J.Sh.F. Tempestade. Em 1818, Fourier investigou a questão das condições de aplicabilidade do método desenvolvido por Newton para a solução numérica de equações, desconhecendo resultados semelhantes obtidos em 1768 pelo matemático francês J.R. Murail. O resultado do trabalho de Fourier sobre métodos numéricos para resolver equações é a Análise de Certas Equações, publicada postumamente em 1831.

A principal área de estudo de Fourier era a física matemática. Em 1807 e 1811 ele apresentou suas primeiras descobertas sobre a teoria da propagação do calor em sólidos para a Academia de Ciências de Paris, e em 1822 publicou o conhecido trabalho Analytical Theory of Heat, que desempenhou um papel importante na história subsequente de matemática. Esta é a teoria matemática da condução de calor. Devido à generalidade do método, este livro tornou-se a fonte de todos os métodos modernos de física matemática. Neste trabalho, Fourier derivou uma equação diferencial para condução de calor e desenvolveu as idéias descritas anteriormente por D. Bernoulli em termos mais gerais, desenvolveu um método de separação de variáveis ​​(método de Fourier) para resolver a equação de calor para certas condições de contorno dadas, que ele aplicou a vários casos especiais (cubo, cilindro, etc.). Este método é baseado na representação de funções por séries trigonométricas de Fourier.

As séries de Fourier tornaram-se agora uma ferramenta bem desenvolvida na teoria de equações diferenciais parciais para resolver problemas de valor de contorno.

1. O conceito de série de Fourier.(pág. 94, Uvarenkov)

As séries de Fourier desempenham um papel importante na física matemática, na teoria da elasticidade, na engenharia elétrica e, especialmente, em seu caso especial - as séries trigonométricas de Fourier.

Uma série trigonométrica é uma série da forma

ou, simbolicamente:

(1)

onde ω, a 0 , a 1 , …, a n , …, b 0 , b 1 , …, b n , … são números constantes (ω>0) .

Alguns problemas da física levaram historicamente ao estudo de tais séries, por exemplo, o problema das vibrações das cordas (século 18), o problema das regularidades nos fenômenos de condução de calor, etc. Em aplicações, consideração de séries trigonométricas , está relacionado principalmente ao problema de representar um dado movimento, descrito pela equação y = ƒ(χ), em

a soma dos mais simples vibrações harmônicas, muitas vezes tomado em um número infinitamente grande, ou seja, como a soma de uma série da forma (1).

Assim, chegamos ao seguinte problema: descobrir se para uma dada função ƒ(x) em um dado intervalo existe uma série (1) que convergiria para essa função nesse intervalo. Se isso for possível, diz-se que a função ƒ(x) se expande em uma série trigonométrica nesse intervalo.

A série (1) converge em algum ponto x 0, devido à periodicidade das funções

(n=1,2,..), também convergirá em todos os pontos da forma (m é qualquer número inteiro), e assim sua soma S(x) será (na região de convergência da série) um periódico função: se S n ( x) é a enésima soma parcial desta série, então temos

e, portanto,

, isto é, S(x 0 +T)=S(x 0). Portanto, falando sobre a expansão de alguma função ƒ(x) em uma série da forma (1), vamos supor que ƒ(x) é uma função periódica.

2. Determinação dos coeficientes da série pelas fórmulas de Fourier.

Seja uma função periódica ƒ(x) com período 2π tal que seja representada por uma série trigonométrica convergindo para uma dada função no intervalo (-π, π), ou seja, é a soma desta série:

. (2)

Suponha que a integral da função do lado esquerdo desta igualdade seja igual à soma das integrais dos termos desta série. Isso será verdade se assumirmos que a série numérica composta pelos coeficientes da série trigonométrica dada converge absolutamente, ou seja, a série numérica positiva converge

(3)

A série (1) é majoritária e pode ser integrada termo a termo no intervalo (-π, π). Integramos ambas as partes da igualdade (2):

.

Calculamos separadamente cada integral que ocorre no lado direito:

, , .

Nesse caminho,

, Onde . (4)

Estimação dos coeficientes de Fourier.(Bugrov)

Teorema 1. Seja uma função ƒ(x) de período 2π ter uma derivada contínua ƒ ( s) (x) pedido s satisfazendo a desigualdade em todo o eixo real:

│ ƒ (s) (x)│≤ M s ; (5)

então os coeficientes de Fourier da função ƒ satisfaça a desigualdade

(6)

Prova. Integrando por partes e levando em consideração que

ƒ(-π) = ƒ(π), temos


Integrando o lado direito de (7) sequencialmente, levando em consideração que as derivadas ƒ ΄ , …, ƒ (s-1) são contínuas e assumem os mesmos valores nos pontos t = -π e t = π, também como estimativa (5), obtemos a primeira estimativa ( 6).

A segunda estimativa (6) é obtida de forma semelhante.

Teorema 2. Os coeficientes de Fourier ƒ(x) satisfazem a desigualdade

(8)

Prova. Nós temos

A série de Fourier é uma representação de uma função tomada arbitrariamente com um período específico como uma série. NO visão geral esta solução é chamada de decomposição de um elemento em uma base ortogonal. A expansão de funções em uma série de Fourier é uma ferramenta bastante poderosa para resolver vários problemas devido às propriedades dessa transformação ao integrar, diferenciar e deslocar uma expressão em um argumento e convolução.

Uma pessoa que não está familiarizada com matemática superior, bem como com os trabalhos do cientista francês Fourier, provavelmente não entenderá o que são essas “séries” e para que servem. Enquanto isso, essa transformação se tornou bastante densa em nossas vidas. Ele é usado não apenas por matemáticos, mas também por físicos, químicos, médicos, astrônomos, sismólogos, oceanógrafos e muitos outros. Vejamos também mais de perto as obras do grande cientista francês, que fez uma descoberta à frente de seu tempo.

O Homem e a Transformada de Fourier

A série de Fourier é um dos métodos (junto com a análise e outros) Esse processo ocorre toda vez que uma pessoa ouve algum som. Nosso ouvido converte automaticamente partículas elementares em um meio elástico são decompostos em linhas (ao longo do espectro) de valores sucessivos do nível de intensidade para tons de diferentes alturas. Em seguida, o cérebro transforma esses dados em sons familiares para nós. Tudo isso acontece em adição ao nosso desejo ou consciência, por si só, mas para entender esses processos, levará vários anos para estudar matemática superior.

Mais sobre a Transformada de Fourier

A transformada de Fourier pode ser realizada por métodos analíticos, numéricos e outros. As séries de Fourier referem-se à maneira numérica de decompor quaisquer processos oscilatórios - desde marés oceânicas e ondas de luz até ciclos de atividade solar (e outros objetos astronômicos). Usando essas técnicas matemáticas, é possível analisar funções, representando quaisquer processos oscilatórios como uma série de componentes senoidais que vão do mínimo ao máximo e vice-versa. A transformada de Fourier é uma função que descreve a fase e a amplitude das senoides correspondentes a uma frequência específica. Este processo pode ser usado para resolver equações muito complexas que descrevem processos dinâmicos que ocorrem sob a influência de energia térmica, luminosa ou elétrica. Além disso, as séries de Fourier permitem isolar os componentes constantes em sinais oscilatórios complexos, o que permitiu interpretar corretamente as observações experimentais obtidas em medicina, química e astronomia.

Referência do histórico

O pai fundador desta teoria é o matemático francês Jean Baptiste Joseph Fourier. Esta transformação foi posteriormente nomeada em sua homenagem. Inicialmente, o cientista aplicou seu método para estudar e explicar os mecanismos de condução de calor - a propagação do calor em sólidos. Fourier sugeriu que a distribuição irregular original pode ser decomposta nas senóides mais simples, cada uma das quais terá sua própria temperatura mínima e máxima, bem como sua própria fase. Nesse caso, cada um desses componentes será medido do mínimo ao máximo e vice-versa. A função matemática que descreve os picos superior e inferior da curva, bem como a fase de cada um dos harmônicos, é chamada de transformada de Fourier da expressão de distribuição de temperatura. O autor da teoria reduziu a função de distribuição geral, que é difícil de descrever matematicamente, a uma série muito conveniente de cosseno e seno, que somam para dar a distribuição original.

O princípio da transformação e as visões dos contemporâneos

Os contemporâneos do cientista - os principais matemáticos do início do século XIX - não aceitaram essa teoria. A principal objeção foi a afirmação de Fourier de que uma função descontínua descrevendo uma linha reta ou uma curva descontínua pode ser representada como uma soma de expressões senoidais que são contínuas. Como exemplo, considere o "passo" de Heaviside: seu valor é zero à esquerda da lacuna e um à direita. Esta função descreve a dependência da corrente elétrica na variável tempo quando o circuito é fechado. Contemporâneos da teoria naquela época nunca haviam encontrado situação similar, quando a expressão descontínua seria descrita por uma combinação de funções contínuas convencionais, como exponencial, senoidal, linear ou quadrática.

O que confundiu os matemáticos franceses na teoria de Fourier?

Afinal, se o matemático estava certo em suas afirmações, então, somando a série de Fourier trigonométrica infinita, pode-se obter uma representação exata da expressão passo a passo, mesmo que tenha muitos passos semelhantes. No início do século XIX, tal afirmação parecia absurda. Mas, apesar de todas as dúvidas, muitos matemáticos ampliaram o escopo do estudo desse fenômeno, levando-o para além do escopo dos estudos de condutividade térmica. No entanto, a maioria dos cientistas continuou a ser atormentada pela pergunta: "A soma de uma série senoidal pode convergir para valor exato função descontínua?"

Convergência da Série de Fourier: Um Exemplo

A questão da convergência é levantada sempre que é necessário somar séries infinitas de números. Para entender esse fenômeno, considere um exemplo clássico. Você pode alcançar a parede se cada passo sucessivo for metade do tamanho do anterior? Suponha que você esteja a dois metros do gol, o primeiro passo o aproxima da metade do caminho, o próximo da marca de três quartos e, após o quinto passo, você percorrerá quase 97% do caminho. No entanto, não importa quantos passos você dê, você não alcançará o objetivo pretendido em um sentido matemático estrito. Usando cálculos numéricos, pode-se mostrar que no final é possível se aproximar de uma distância dada arbitrariamente pequena. Esta prova equivale a demonstrar que o valor total de metade, um quarto, etc. tenderá a um.

Uma Questão de Convergência: A Segunda Vinda, ou o Aparelho de Lorde Kelvin

essa questão surgiu no final do século XIX, quando se tentou usar as séries de Fourier para prever a intensidade do fluxo e refluxo. Neste momento, um dispositivo foi inventado por Lord Kelvin, que é um dispositivo de computação analógica que permitia aos marinheiros da frota militar e mercante rastrear este um fenômeno natural. Esse mecanismo determinava os conjuntos de fases e amplitudes a partir de uma tabela de alturas de maré e seus momentos de tempo correspondentes, cuidadosamente medidos em um determinado porto durante o ano. Cada parâmetro era um componente senoidal da expressão da altura da maré e era um dos componentes regulares. Os resultados das medições foram inseridos na calculadora de Lord Kelvin, que sintetizou uma curva que previa a altura da água em função do tempo para o próximo ano. Muito em breve curvas semelhantes foram traçadas para todos os portos do mundo.

E se o processo for interrompido por uma função descontínua?

Naquela época, parecia óbvio que um preditor de ondas de maré com grande quantidade elementos da conta podem calcular um grande número de fases e amplitudes e assim fornecer previsões mais precisas. No entanto, verificou-se que essa regularidade não é observada naqueles casos em que a expressão de maré a ser sintetizada continha um salto acentuado, ou seja, era descontínua. Caso os dados sejam inseridos no dispositivo a partir da tabela de momentos de tempo, ele calcula vários coeficientes de Fourier. A função original é restaurada graças aos componentes senoidais (de acordo com os coeficientes encontrados). A discrepância entre a expressão original e a restaurada pode ser medida em qualquer ponto. Ao realizar cálculos e comparações repetidos, pode-se observar que o valor do maior erro não diminui. No entanto, eles estão localizados na região correspondente ao ponto de descontinuidade e tendem a zero em qualquer outro ponto. Em 1899, este resultado foi teoricamente confirmado por Joshua Willard Gibbs da Universidade de Yale.

Convergência das séries de Fourier e o desenvolvimento da matemática em geral

A análise de Fourier não é aplicável a expressões contendo um número infinito de rajadas em um determinado intervalo. Em geral, a série de Fourier, se a função original é representada pelo resultado de uma função real dimensão física, sempre convergem. Questões sobre a convergência desse processo para classes específicas de funções levaram ao surgimento de novas seções em matemática, por exemplo, a teoria das funções generalizadas. Está associado a nomes como L. Schwartz, J. Mikusinsky e J. Temple. Dentro da estrutura desta teoria, uma clara e precisa base teórica sob expressões como a função delta de Dirac (descreve uma região de uma única área concentrada em uma vizinhança infinitesimal de um ponto) e o "passo" de Heaviside. Graças a este trabalho, a série de Fourier tornou-se aplicável à resolução de equações e problemas em que aparecem conceitos intuitivos: uma carga pontual, uma massa pontual, dipolos magnéticos e também uma carga concentrada em uma viga.

Método de Fourier

As séries de Fourier, de acordo com os princípios da interferência, começam com a decomposição de formas complexas em formas mais simples. Por exemplo, uma mudança no fluxo de calor é explicada por sua passagem através de vários obstáculos de um material isolante de calor. forma irregular ou uma mudança na superfície da terra - um terremoto, uma mudança na órbita de um corpo celeste - a influência dos planetas. Como regra, equações semelhantes que descrevem sistemas clássicos simples são resolvidas elementarmente para cada onda individual. Fourier mostrou que soluções simples também podem ser resumidas para obter soluções para problemas mais complexos. Expressa na linguagem da matemática, a série de Fourier é uma técnica para representar uma expressão como a soma de harmônicos - cosseno e senoides. É por isso esta análise também conhecido como "análise harmônica".

Série de Fourier - a técnica ideal antes da "era do computador"

Antes da criação da tecnologia computacional, a técnica de Fourier era a melhor arma no arsenal dos cientistas ao trabalhar com a natureza ondulatória do nosso mundo. A série de Fourier em forma complexa permite resolver não só tarefas simples, que são passíveis de aplicação direta das leis da mecânica newtoniana, mas também equações fundamentais. A maioria das descobertas da ciência newtoniana no século XIX só foi possível pela técnica de Fourier.

Série Fourier hoje

Com o desenvolvimento dos computadores, as transformadas de Fourier subiram para um nível qualitativamente novo. Esta técnica está firmemente enraizada em quase todas as áreas da ciência e tecnologia. Um exemplo é um sinal de áudio e vídeo digital. Sua realização só foi possível graças à teoria desenvolvida por um matemático francês no início do século XIX. Assim, a série de Fourier em forma complexa possibilitou um avanço no estudo espaço sideral. Além disso, isso influenciou o estudo da física de materiais semicondutores e plasma, acústica de microondas, oceanografia, radar e sismologia.

Série trigonométrica de Fourier

Em matemática, uma série de Fourier é uma forma de representar funções complexas a soma dos mais simples. Em casos gerais, o número de tais expressões pode ser infinito. Além disso, quanto mais seu número for levado em consideração no cálculo, mais preciso será o resultado final. Mais frequentemente usado como o mais simples funções trigonométricas cosseno ou seno. Neste caso, as séries de Fourier são chamadas de trigonométricas, e a solução de tais expressões é chamada de expansão do harmônico. Este método desempenha um papel importante na matemática. Em primeiro lugar, a série trigonométrica fornece um meio para a imagem, assim como o estudo de funções, é o principal aparato da teoria. Além disso, permite resolver uma série de problemas de física matemática. Finalmente, esta teoria contribuiu para o desenvolvimento e deu vida a uma série de seções muito importantes ciência matemática(a teoria das integrais, a teoria das funções periódicas). Além disso, serviu como ponto de partida para o desenvolvimento das seguintes funções de uma variável real, e também marcou o início da análise harmônica.

Que já estão bastante fartos. E sinto que chegou o momento em que é hora de extrair novos alimentos enlatados das reservas estratégicas da teoria. É possível expandir a função em uma série de alguma outra maneira? Por exemplo, para expressar um segmento de reta em termos de senos e cossenos? Parece incrível, mas essas funções aparentemente distantes se prestam a
"reunião". Além dos graus familiares em teoria e prática, existem outras abordagens para expandir uma função em uma série.

Nesta lição, nos familiarizaremos com a série trigonométrica de Fourier, abordaremos a questão de sua convergência e soma e, é claro, analisaremos vários exemplos para expandir funções em uma série de Fourier. Eu sinceramente queria chamar o artigo de “Série de Fourier para Leigos”, mas isso seria astuto, já que a resolução de problemas exigirá conhecimento de outras seções de análise matemática e alguma experiência prática. Portanto, o preâmbulo vai se assemelhar ao treinamento dos astronautas =)

Primeiro, o estudo dos materiais da página deve ser abordado em excelente forma. Sono, descansado e sóbrio. Sem fortes emoções sobre uma perna quebrada de hamster e pensamentos obsessivos sobre as dificuldades da vida peixes de aquário. A série de Fourier não é difícil em termos de compreensão, no entanto, as tarefas práticas requerem simplesmente concentração aumentada atenção - idealmente, você deve abandonar completamente os estímulos externos. A situação é agravada pelo fato de não haver uma maneira fácil de verificar a solução e a resposta. Assim, se sua saúde estiver abaixo da média, é melhor fazer algo mais simples. Verdade.

Em segundo lugar, antes de voar para o espaço, é necessário estudar o painel de instrumentos da espaçonave. Vamos começar pelos valores das funções que devem ser clicadas na máquina:

Para qualquer valor natural:

1) . E, de fato, a senóide "pisca" o eixo x através de cada "pi":
. No caso de valores negativos do argumento, o resultado, claro, será o mesmo: .

2). Mas nem todos sabiam disso. O cosseno "pi en" é o equivalente a uma "luz intermitente":

Um argumento negativo não altera o caso: .

Talvez o suficiente.

E em terceiro lugar, querido corpo de cosmonautas, você precisa ser capaz de ... integrar.
Em particular, com certeza trazer uma função sob um sinal diferencial, integrar por partes e estar em boas relações com Fórmula de Newton-Leibniz. Vamos começar os importantes exercícios pré-voo. Eu fortemente não recomendo ignorá-lo, para que mais tarde você não achate em gravidade zero:

Exemplo 1

Calcular integrais definidas

onde assume valores naturais.

Solução: a integração é realizada sobre a variável "x" e nesta fase a variável discreta "en" é considerada uma constante. Em todas as integrais trazer a função sob o sinal do diferencial:

Uma versão curta da solução, que seria boa para atirar, se parece com isso:

Acostumando:

Os quatro pontos restantes estão por conta própria. Tente tratar a tarefa conscientemente e organizar as integrais de forma curta. Exemplos de soluções no final da lição.

Depois de um exercício de QUALIDADE, vestimos trajes espaciais
e se preparando para começar!

Expansão de uma função em uma série de Fourier no intervalo

Vamos considerar uma função que determinado pelo menos no intervalo (e, possivelmente, em um intervalo maior). Se esta função é integrável no segmento , então ela pode ser expandida em uma função trigonométrica Séries de Fourier:
, onde estão os chamados Coeficientes de Fourier.

Neste caso, o número é chamado período de decomposição, e o número é decomposição de meia-vida.

É óbvio que em caso Geral A série de Fourier consiste em senos e cossenos:

De fato, vamos escrevê-lo em detalhes:

O termo zero da série é geralmente escrito como .

Os coeficientes de Fourier são calculados usando as seguintes fórmulas:

Entendo perfeitamente que novos termos ainda são obscuros para os iniciantes estudarem o tema: período de decomposição, meio ciclo, Coeficientes de Fourier e outros. Não entre em pânico, não é comparável à excitação antes de uma caminhada espacial. Vamos descobrir tudo no exemplo mais próximo, antes de executar o que é lógico nos perguntar urgente assuntos práticos:

O que você precisa fazer nas seguintes tarefas?

Expanda a função em uma série de Fourier. Além disso, muitas vezes é necessário desenhar um gráfico de uma função, um gráfico da soma de uma série, uma soma parcial e, no caso de fantasias de professores sofisticados, fazer outra coisa.

Como expandir uma função em uma série de Fourier?

Essencialmente, você precisa encontrar Coeficientes de Fourier, ou seja, componha e calcule três integrais definidas.

Por favor, copie a forma geral da série de Fourier e as três fórmulas de trabalho em seu caderno. Estou muito feliz que alguns dos visitantes do site tenham um sonho de infância de se tornar um astronauta se tornando realidade bem na frente dos meus olhos =)

Exemplo 2

Expanda a função em uma série de Fourier no intervalo . Construir um gráfico, um gráfico da soma de uma série e uma soma parcial.

Solução: a primeira parte da tarefa é expandir a função em uma série de Fourier.

O início é padrão, certifique-se de anotar que:

Neste problema, o período de expansão , meio período .

Expandimos a função em uma série de Fourier no intervalo:

Usando as fórmulas apropriadas, encontramos Coeficientes de Fourier. Agora precisamos compor e calcular três integrais definidas. Por conveniência, numerarei os pontos:

1) A primeira integral é a mais simples, porém, já requer um olho e um olho:

2) Usamos a segunda fórmula:

Esta integral é bem conhecida e ele toma aos poucos:

Quando encontrado usado método de trazer uma função sob um sinal diferencial.

Na tarefa em consideração, é mais conveniente usar imediatamente fórmula para integração por partes em uma integral definida :

Algumas notas técnicas. Primeiro, depois de aplicar a fórmula a expressão inteira deve ser colocada entre colchetes grandes, uma vez que existe uma constante na frente da integral original. Não vamos perdê-lo! Os parênteses podem ser abertos em qualquer etapa posterior, eu fiz isso no último turno. Na primeira "peça" mostramos extrema precisão na substituição, como você pode ver, a constante está fora do negócio, e os limites de integração são substituídos no produto. Esta ação está marcada com colchetes. Bem, a integral da segunda "peça" da fórmula é bem conhecida por você da tarefa de treinamento ;-)

E o mais importante - a concentração final de atenção!

3) Estamos procurando o terceiro coeficiente de Fourier:

Uma relativa da integral anterior é obtida, que também é integrado por partes:

Esta instância é um pouco mais complicada, vou comentar os próximos passos passo a passo:

(1) A expressão inteira é colocada entre colchetes grandes.. Eu não queria parecer um chato, eles perdem a constante com muita frequência.

(2) B este caso Eu imediatamente abri aqueles grandes colchetes. Atenção especial dedicamos à primeira “peça”: a constante fuma à margem e não participa da substituição dos limites de integração ( e ) no produto . Em vista da desordem do registro, é novamente aconselhável destacar esta ação entre colchetes. Com a segunda "peça" tudo é mais simples: aqui a fração apareceu depois de abrir grandes colchetes e a constante - como resultado da integração da integral familiar ;-)

(3) Entre colchetes, realizamos transformações e, na integral à direita, substituímos os limites de integração.

(4) Retiramos o “piscador” dos colchetes: , após o que abrimos os colchetes internos: .

(5) Cancelamos 1 e -1 entre parênteses, fazemos simplificações finais.

Finalmente encontrei todos os três coeficientes de Fourier:

Substitua-os na fórmula :

Não se esqueça de dividir ao meio. Na última etapa, a constante ("menos dois"), que não depende de "en", é retirada da soma.

Assim, obtivemos a expansão da função em uma série de Fourier no intervalo:

Vamos estudar a questão da convergência da série de Fourier. Vou explicar a teoria em particular Teorema de Dirichlet, literalmente "nos dedos", então se você precisar de formulações estritas, consulte um livro-texto sobre cálculo (por exemplo, o 2º volume de Bohan; ou o 3º volume de Fichtenholtz, mas é mais difícil nele).

Na segunda parte da tarefa, é necessário desenhar um gráfico, um gráfico de soma em série e um gráfico de soma parcial.

O gráfico da função é o usual linha reta no avião, que é desenhado com uma linha pontilhada preta:

Nós lidamos com a soma da série. Como você sabe, séries funcionais convergem para funções. No nosso caso, a série de Fourier construída para qualquer valor de "x" converge para a função mostrada em vermelho. Esta função está sujeita a quebras de 1º tipo em pontos, mas também definidos neles (pontos vermelhos no desenho)

Nesse caminho: . É fácil ver que ela difere marcadamente da função original, razão pela qual na notação um til é usado em vez de um sinal de igual.

Vamos estudar um algoritmo pelo qual é conveniente construir a soma de uma série.

No intervalo central, a série de Fourier converge para a própria função (o segmento central vermelho coincide com a linha pontilhada preta da função linear).

Agora vamos falar um pouco sobre a natureza da expansão trigonométrica considerada. Séries de Fourier inclui apenas funções periódicas (constante, senos e cossenos), então a soma da série também é uma função periódica.

O que isso significa em nosso exemplo específico? E isso significa que a soma da série necessariamente periódica e o segmento vermelho do intervalo deve ser repetido infinitamente à esquerda e à direita.

Acho que agora o significado da frase "período de decomposição" finalmente ficou claro. Simplificando, toda vez que a situação se repete de novo e de novo.

Na prática, geralmente é suficiente representar três períodos de decomposição, como é feito no desenho. Bem, e mais "tocos" de períodos vizinhos - para deixar claro que o gráfico continua.

De particular interesse são pontos de descontinuidade do 1º tipo. Nesses pontos, a série de Fourier converge para valores isolados, que se localizam exatamente no meio do "salto" da descontinuidade (pontos vermelhos no desenho). Como encontrar a ordenada desses pontos? Primeiro, vamos encontrar a ordenada do "piso superior": para isso, calculamos o valor da função no ponto mais à direita do período de expansão central: . Para calcular a ordenada do “piso inferior”, a maneira mais fácil é pegar o valor mais à esquerda do mesmo período: . A ordenada do valor médio é a média aritmética da soma do "superior e inferior": . Agradável é o fato de que, ao construir um desenho, você verá imediatamente se o meio está calculado corretamente ou incorretamente.

Vamos construir uma soma parcial da série e ao mesmo tempo repetir o significado do termo "convergência". O motivo é conhecido da lição sobre a soma da série numérica. Vamos descrever nossa riqueza em detalhes:

Para fazer uma soma parcial, você precisa escrever zero + mais dois termos da série. Aquilo é,

No desenho, o gráfico da função é mostrado em verde e, como você pode ver, envolve a soma total com bastante força. Se considerarmos uma soma parcial de cinco termos da série, o gráfico dessa função aproximará as linhas vermelhas com ainda mais precisão; se houver cem termos, a “serpente verde” se fundirá completamente com os segmentos vermelhos, etc. Assim, a série de Fourier converge para sua soma.

É interessante notar que qualquer soma parcial é função contínua, mas a soma total da série ainda é descontínua.

Na prática, não é incomum construir um gráfico de soma parcial. Como fazer isso? No nosso caso, é necessário considerar a função no segmento, calcular seus valores nas extremidades do segmento e em pontos intermediários (quanto mais pontos você considerar, mais preciso será o gráfico). Em seguida, você deve marcar esses pontos no desenho e desenhar cuidadosamente um gráfico no período e, em seguida, “replicá-lo” em intervalos adjacentes. De que outra forma? Afinal, a aproximação também é uma função periódica... ...seu gráfico de alguma forma me lembra um ritmo cardíaco regular no visor de um dispositivo médico.

Obviamente, não é muito conveniente realizar a construção, pois você deve ser extremamente cuidadoso, mantendo uma precisão não inferior a meio milímetro. No entanto, vou agradar os leitores que estão em desacordo com o desenho - em uma tarefa "real", está longe de ser sempre necessário realizar um desenho, em algum lugar em 50% dos casos é necessário expandir a função em uma série de Fourier e isso é isto.

Depois de concluir o desenho, concluímos a tarefa:

Responda:

Em muitas tarefas, a função sofre ruptura do 1º tipo logo no período de decomposição:

Exemplo 3

Expanda em uma série de Fourier a função dada no intervalo . Desenhe um gráfico da função e a soma total da série.

A função proposta é dada por partes (e, lembre-se, apenas no segmento) e suportar ruptura do 1º tipo no ponto . É possível calcular os coeficientes de Fourier? Sem problemas. As partes esquerda e direita da função são integráveis ​​em seus intervalos, de modo que as integrais em cada uma das três fórmulas devem ser representadas como a soma de duas integrais. Vejamos, por exemplo, como isso é feito para um coeficiente zero:

A segunda integral acabou sendo igual a zero, o que reduziu o trabalho, mas nem sempre é assim.

Dois outros coeficientes de Fourier são escritos de forma semelhante.

Como exibir a soma de uma série? No intervalo esquerdo, desenhamos um segmento de linha reta e no intervalo - um segmento de linha reta (destaque a seção do eixo em negrito). Ou seja, no intervalo de expansão, a soma da série coincide com a função em todos os lugares, exceto por três pontos "ruins". No ponto de descontinuidade da função, a série de Fourier converge para um valor isolado, que está localizado exatamente no meio do “salto” da descontinuidade. Não é difícil vê-lo oralmente: limite esquerdo:, limite direito: e, obviamente, a ordenada do ponto médio é 0,5.

Devido à periodicidade da soma , a imagem deve ser “multiplicada” em períodos vizinhos, em particular, retratar a mesma coisa nos intervalos e . Neste caso, nos pontos, a série de Fourier converge para os valores medianos.

Na verdade, não há nada de novo aqui.

Tente resolver esse problema sozinho. Uma amostra aproximada de bom design e desenho no final da lição.

Expansão de uma função em uma série de Fourier em um período arbitrário

Para um período de expansão arbitrário, onde "el" é qualquer número positivo, as fórmulas para a série de Fourier e coeficientes de Fourier diferem em um argumento de seno e cosseno um pouco mais complicado:

Se , então obtemos as fórmulas para o intervalo com o qual começamos.

O algoritmo e os princípios para resolver o problema são completamente preservados, mas a complexidade técnica dos cálculos aumenta:

Exemplo 4

Expanda a função em uma série de Fourier e plote a soma.

Solução: de fato, um análogo do Exemplo nº 3 com ruptura do 1º tipo no ponto . Neste problema, o período de expansão , meio período . A função é definida apenas no half-interval , mas isso não muda as coisas - é importante que ambas as partes da função sejam integráveis.

Vamos expandir a função em uma série de Fourier:

Como a função é descontínua na origem, cada coeficiente de Fourier deve obviamente ser escrito como a soma de duas integrais:

1) Vou escrever a primeira integral o mais detalhada possível:

2) Observe cuidadosamente a superfície da lua:

Segunda integral levar em partes:

O que você deve prestar atenção depois de abrirmos a continuação da solução com um asterisco?

Primeiro, não perdemos a primeira integral , onde executamos imediatamente trazendo sob o signo do diferencial. Em segundo lugar, não se esqueça da constante malfadada antes dos grandes colchetes e não se confunda com os sinais ao usar a fórmula . Suportes grandes, afinal, é mais conveniente abrir imediatamente na próxima etapa.

O resto é uma questão de técnica, apenas a experiência insuficiente na resolução de integrais pode causar dificuldades.

Sim, não foi em vão que os eminentes colegas do matemático francês Fourier ficaram indignados - como ele se atreveu a decompor funções em séries trigonométricas ?! =) A propósito, provavelmente todos estão interessados ​​no significado prático da tarefa em questão. O próprio Fourier trabalhou modelo matemático condutividade térmica, e mais tarde a série que leva seu nome começou a ser usada para estudar muitos processos periódicos, que são aparentemente invisíveis no mundo circundante. Agora, a propósito, me peguei pensando que não foi por acaso que comparei o gráfico do segundo exemplo com um ritmo cardíaco periódico. Os interessados ​​podem conhecer a aplicação prática Transformadas de Fourier de fontes de terceiros. ... Embora seja melhor não - será lembrado como Primeiro Amor =)

3) Dados os elos fracos repetidamente mencionados, lidamos com o terceiro coeficiente:

Integrando por partes:

Substituímos os coeficientes de Fourier encontrados na fórmula , não esquecendo de dividir o coeficiente zero pela metade:

Vamos traçar a soma da série. Vamos repetir brevemente o procedimento: no intervalo construímos uma linha e no intervalo - uma linha. Com um valor zero de "x", colocamos um ponto no meio do "salto" do gap e "replicamos" o gráfico para períodos vizinhos:


Nas "junções" dos períodos, a soma também será igual aos pontos médios do "salto" do gap.

Preparar. Lembro que a função em si é condicionalmente definida apenas no meio-intervalo e, obviamente, coincide com a soma das séries nos intervalos

Responda:

Às vezes, uma função dada por partes também é contínua no período de expansão. O exemplo mais simples: . Solução (Veja Bohan Volume 2)é o mesmo que nos dois exemplos anteriores: apesar continuidade de função no ponto , cada coeficiente de Fourier é expresso como a soma de duas integrais.

No intervalo de separação pontos de descontinuidade do 1º tipo e/ou pontos de "junção" do gráfico podem ser mais (dois, três e, em geral, qualquer final quantia). Se uma função é integrável em todas as partes, ela também é expansível em uma série de Fourier. Mas, por experiência prática, não me lembro de tal lata. No entanto, existem tarefas mais difíceis do que apenas consideradas, e no final do artigo para todos há links para séries de Fourier de maior complexidade.

Enquanto isso, vamos relaxar, recostando-nos em nossas cadeiras e contemplando as infinitas extensões de estrelas:

Exemplo 5

Expanda a função em uma série de Fourier no intervalo e plote a soma da série.

Nesta tarefa, a função contínuo no meio-intervalo de decomposição, o que simplifica a solução. Tudo é muito semelhante ao Exemplo #2. Você não pode fugir da nave espacial - você terá que decidir =) Modelo de amostra no final da lição, a programação está em anexo.

Expansão em série de Fourier de funções pares e ímpares

Com funções pares e ímpares, o processo de resolução do problema é visivelmente simplificado. E é por isso. Vamos retornar à expansão da função em uma série de Fourier em um período de "dois pi" e período arbitrário "duas cervejas" .

Vamos supor que nossa função seja par. O termo geral da série, como você pode ver, contém cossenos pares e senos ímpares. E se decompusermos uma função EVEN, então por que precisamos de senos ímpares?! Vamos redefinir o coeficiente desnecessário: .

Nesse caminho, uma função par se expande em uma série de Fourier apenas em cossenos:

Porque o integrais de funções pares sobre um segmento de integração simétrico em relação a zero pode ser duplicado, então o resto dos coeficientes de Fourier também são simplificados.

Para intervalo:

Para um intervalo arbitrário:

Exemplos de livros didáticos encontrados em quase todos os livros didáticos de cálculo incluem expansões de funções pares . Além disso, eles se encontraram repetidamente em minha prática pessoal:

Exemplo 6

Dada uma função. Requeridos:

1) expanda a função em uma série de Fourier com período , onde é um número positivo arbitrário;

2) escreva a expansão no intervalo , construa uma função e represente graficamente a soma total da série .

Solução: no primeiro parágrafo, propõe-se resolver o problema de maneira geral, e isso é muito conveniente! Haverá uma necessidade - basta substituir o seu valor.

1) Neste problema, o período de expansão , meio período . No decurso de outras ações, em particular durante a integração, "el" é considerado uma constante

A função é par, o que significa que ela se expande em uma série de Fourier apenas em cossenos: .

Os coeficientes de Fourier são procurados pelas fórmulas . Preste atenção às suas vantagens absolutas. Primeiro, a integração é realizada sobre o segmento positivo da expansão, o que significa que nos livramos do módulo com segurança , considerando apenas "x" de duas peças. E, em segundo lugar, a integração é visivelmente simplificada.

Dois:

Integrando por partes:

Nesse caminho:
, enquanto a constante , que não depende de "en", é retirada da soma.

Responda:

2) Escrevemos a expansão no intervalo, para isso substituímos o valor desejado do meio período na fórmula geral:

Expansão em série de Fourier de funções pares e ímpares Expansão de uma função dada em um segmento em uma série em termos de senos ou cossenos Série de Fourier para uma função com um período arbitrário Representação complexa da série de Fourier Série de Fourier em geral sistemas ortogonais de funções Série de Fourier em um sistema ortogonal Propriedade mínima dos coeficientes de Fourier Desigualdade de Bessel Igualdade Parseval Sistemas fechados Completude e fechamento de sistemas


Expansão em série de Fourier de funções pares e ímpares A função f(x), definida no segmento \-1, onde I > 0, é chamada par se o gráfico da função par for simétrico em relação ao eixo y. A função f(x) definida no segmento J, onde I > 0, é chamada ímpar se o Gráfico da função ímpar for simétrico em relação à origem. Exemplo. a) A função é par no segmento |-jt, jt), pois para todo x e b) A função é ímpar, pois a expansão em série de Fourier de funções pares e ímpares é a expansão de uma função dada no segmento em uma série de senos ou cossenos Série de Fourier para uma função com um período arbitrário Notação complexa da série de Fourier Série de Fourier em sistemas ortogonais gerais de funções Série de Fourier em um sistema ortogonal Propriedade mínima dos coeficientes de Fourier Desigualdade de Bessel Igualdade de Parseval Sistemas fechados Completude e fechamento de sistemas c) Função f(x)=x2-x, onde não pertence nem a funções pares nem a funções ímpares, pois Seja a função f(x) que satisfaz as condições do Teorema 1 ser par no segmento x|. Então, para todos, ou seja, /(g) cos nx é uma função par, e f(x)sinnx é uma função ímpar. Portanto, os coeficientes de Fourier de uma função par /(x) serão iguais, portanto, a série de Fourier de uma função par tem a forma f(x) sen nx é uma função par. Portanto, teremos Assim, a série de Fourier de uma função ímpar tem a forma Temos Aplicando integração por partes duas vezes, obtemos que Assim, a série de Fourier desta função fica assim: ou, na forma expandida, Esta igualdade é válida para qualquer x €, pois nos pontos x = ±ir a soma dos series coincide com os valores da função f(x ) = x2, pois os gráficos da função f(x) = x e as somas das séries resultantes são dadas na fig. Comente. Esta série de Fourier permite encontrar a soma de uma das séries numéricas convergentes, ou seja, para x \u003d 0, obtemos que A função /(x) satisfaz as condições do Teorema 1, portanto pode ser expandida em uma série de Fourier, que, devido à estranheza desta função, terá a forma Integrando por partes, encontramos os coeficientes de Fourier. série desta função tem a forma Esta igualdade vale para todos os pontos x В x - ±tg a soma da série de Fourier não coincide com os valores da função / (x) = x, pois é igual a Fora do segmento [- *, n-] a soma da série é uma continuação periódica da função / (x) \u003d x; seu gráfico é mostrado na Fig. 6. § 6. Expansão de uma função dada em um intervalo em uma série em termos de senos ou cossenos Seja uma função monotônica limitada por partes / seja dada em um intervalo . Os valores desta função no intervalo 0| pode ser definida de várias maneiras. Por exemplo, é possível definir a função / no segmento mc] de tal forma que /. Neste caso diz-se que) "se estende ao segmento 0] de forma par"; sua série de Fourier conterá apenas cossenos. Se, no entanto, a função /(x) é definida no segmento [-x, mc] de modo que /(, então uma função ímpar é obtida, e então dizemos que / "é estendido para o segmento [-*, 0 ] de uma maneira ímpar"; neste caso, a série de Fourier conterá apenas senos. senos e cossenos.Exemplo 1. Expanda a função em uma série de Fourier: a) por cossenos; b) ao longo dos senos. M Esta função, com suas extensões pares e ímpares para o segmento |-x, 0) será limitada e monotônica por partes. a) Continuamos / (z) no segmento 0) a) Continuamos j \ x) no segmento (-m, 0 | de maneira par (Fig. 7), então sua série de Fourier i terá a forma P \u003d 1 onde os coeficientes de Fourier são iguais, respectivamente para Portanto, b) Vamos continuar /(z) no segmento [-x,0] de forma ímpar (Fig. 8). Em seguida, sua série de Fourier §7. Série de Fourier para uma função com um período arbitrário Seja a função fix) periódica com um período de 21,1 ^ 0. Para expandi-la em uma série de Fourier no intervalo onde I > 0, fazemos uma mudança de variável definindo x = jt . Então a função F(t) = / ^tj será uma função periódica do argumento t com um período e pode ser expandida em um segmento de uma série de Fourier Voltando à variável x, ou seja, definindo, obtemos , permanecemos em força também para funções periódicas com um período arbitrário 21. Em particular, o critério suficiente para a expansão de uma função em uma série de Fourier também permanece válido. Exemplo 1. Expanda em uma série de Fourier uma função periódica com um período de 21, dado no segmento [-/,/] pela fórmula (Fig. 9). Como essa função é par, sua série de Fourier tem a forma Substituindo os valores encontrados dos coeficientes de Fourier na série de Fourier, obtemos Notamos uma propriedade importante das funções periódicas. Teorema 5. Se uma função tem um período T e é integrável, então para qualquer número a a igualdade m é válida. ou seja, a integral em um segmento cujo comprimento é igual ao período T tem o mesmo valor independentemente da posição desse segmento no eixo real. De fato, fazemos uma mudança de variável na segunda integral, assumindo Isto dá e, portanto, Geometricamente, esta propriedade significa que no caso da área sombreada na Fig. 10 áreas são iguais entre si. Em particular, para uma função f(x) com um período, obtemos na expansão em série de Fourier de funções pares e ímpares a expansão de uma função dada em um segmento em uma série em termos de senos ou cossenos Série de Fourier para uma função com um período arbitrário Representação complexa da série de Fourier Série de Fourier em funções gerais de sistemas ortogonais Série de Fourier em um sistema ortogonal Propriedade mínima dos coeficientes de Fourier Desigualdade de Bessel Igualdade de Parseval Sistemas fechados Completude e fechamento de sistemas que os coeficientes de Fourier de uma função periódica f(x) com um período de 21 pode ser calculado usando as fórmulas onde a é um número real(note que as funções cos - e sin têm um período de 2/). Exemplo 3. Expanda em uma série de Fourier uma função dada em um intervalo com um período de 2x (Fig. 11). 4 Encontre os coeficientes de Fourier desta função. Colocando as fórmulas encontramos que para Portanto, a série de Fourier ficará assim: No ponto x = jt (ponto de descontinuidade do primeiro tipo) temos §8. Notação complexa da série de Fourier Nesta seção, alguns elementos são usados análise complexa(Consulte o capítulo XXX, onde todas as operações aqui realizadas com expressões complexas, são estritamente justificadas). Deixe que a função f(x) satisfaça condições suficientes para expansão em uma série de Fourier. Então no segmento x] ele pode ser representado por uma série da forma Usando as fórmulas de Euler Substituindo essas expressões na série (1) em vez de cos nx e sen xy teremos Introduzimos a seguinte notação Então a série (2) toma a forma Assim, a série de Fourier (1) é apresentada na forma complexa (3). Vamos encontrar expressões para os coeficientes em termos de integrais. Temos Da mesma forma, encontramos Finalmente, as fórmulas para с„, с_п e с podem ser escritas da seguinte forma: . . Os coeficientes cn são chamados de coeficientes complexos de Fourier da função Para uma função periódica com um período), a forma complexa da série de Fourier assume a forma dado valor f, se houver limites Exemplo. Expanda a função de período em uma série de Fourier complexa Esta função satisfaz condições suficientes para expansão em uma série de Fourier. Vamos Encontrar os coeficientes de Fourier complexos desta função. Temos para ímpar para n par, ou, em suma. Substituindo os valores), obtemos finalmente Note que esta série também pode ser escrita da seguinte forma: Série de Fourier em sistemas ortogonais gerais de funções 9.1. Sistemas ortogonais de funções Denote pelo conjunto de todas as funções (reais) que são definidas ao quadrado e integráveis ​​no intervalo [a, 6], isto é, aquelas para as quais existe uma integral. Em particular, todas as funções f(x) que são contínuas no intervalo [a , 6], pertencem a 6], e os valores de suas integrais de Lebesgue coincidem com os valores das integrais de Riemann. Definição. O sistema de funções, onde, é chamado ortogonal no intervalo [a, b\, se a Condição (1) assumir, em particular, que nenhuma das funções é identicamente igual a zero. A integral é entendida no sentido de Lebesgue. e chamamos a quantidade de norma de uma função.Se em um sistema ortogonal para qualquer n que temos, então o sistema de funções é chamado ortonormal. Se o sistema (y>n(x)) é ortogonal, então o sistema Exemplo 1. Um sistema trigonométrico é ortogonal em um segmento. O sistema de funções é um sistema ortonormal de funções no Exemplo 2. O sistema cosseno e o sistema seno é ortonormal. Vamos introduzir a notação de que elas são ortogonais no segmento (0, f|, mas não ortonormais (para I ↦ 2). Como suas normas são COS que as funções formam um sistema ortonormal de funções em um segmento. Vamos mostrar, por exemplo, que os polinômios de Legendre são ortogonais. Seja m > n. Neste caso, integrando n vezes por partes, encontramos, pois para a função t/m = (z2 - I)m, todas as derivadas até a ordem m - Eu inclusive desapareço nas extremidades do intervalo [-1,1). Definição. O sistema de funções (pn(x)) é chamado ortogonal no intervalo (a, b) por saliência p(x) se: 1) existem integrais para todo n = 1,2,... Aqui assume-se que a função peso p(x) é definida e positiva em todo o intervalo (a, b), com a possível exceção de um número finito de pontos onde p(x) pode desaparecer. Após realizar a diferenciação na fórmula (3), encontramos. Pode-se mostrar que os polinômios de Chebyshev-Hermite são ortogonais no intervalo Exemplo 4. O sistema de funções de Bessel (jL(pix)^ é ortogonal no intervalo de zeros da função de Bessel Exemplo 5. Considere os polinômios de Chebyshev-Hermite, que pode ser definida usando igualdade. Série de Fourier em um sistema ortogonal Seja um sistema ortogonal de funções no intervalo (a, 6) e deixe a série (cj = const) convergir nesse intervalo para a função f(x): Multiplicando ambos os lados da última igualdade por - fixo) e integrando sobre x de a até 6, devido à ortogonalidade do sistema, obtemos que Esta operação tem, em geral, um caráter puramente formal. No entanto, em alguns casos, por exemplo, quando a série (4) converge uniformemente, todas as funções são contínuas e o intervalo (a, 6) é finito, esta operação é válida. Mas é a interpretação formal que é importante para nós agora. Então, digamos que uma função é dada. Formamos os números c * de acordo com a fórmula (5) e escrevemos A série do lado direito é chamada de série de Fourier da função f (x) em relação ao sistema (^n (n)) - Os números Cn são chamados coeficientes de Fourier da função f(x) neste sistema. O sinal ~ na fórmula (6) significa apenas que os números Cn estão relacionados com a função f(x) pela fórmula (5) (neste caso, não é assumido que a série à direita converge, muito menos converge para a função f(x)). Portanto, surge naturalmente a pergunta: quais são as propriedades desta série? Em que sentido ela "representa" a função f(x)? 9.3. Definição de convergência média. Uma sequência converge para um elemento ] em média se a norma está no espaço Teorema 6. Se uma sequência ) converge uniformemente, então também converge em média. M Deixe a sequência ()) convergir uniformemente no segmento [a, b] para a função f(x). Isso significa que para qualquer, para todo n suficientemente grande, temos Daí, do qual segue nossa afirmação. A recíproca não é verdadeira: a sequência () pode convergir em média para /(x), mas não ser uniformemente convergente. Exemplo. Consideremos a sequência nx É fácil ver que Mas esta convergência não é uniforme: existe e, por exemplo, tal que não importa quão grande seja n, no segmento Série de Fourier para uma função com um período arbitrário Representação complexa de a série de Fourier Série de Fourier em geral sistemas ortogonais de funções Série de Fourier em um sistema ortogonal Propriedade mínima dos coeficientes de Fourier Desigualdade de Bessel Igualdade de Parseval Sistemas fechados Completude e fechamento de sistemas e let ) no sistema ortonormal b Considere uma combinação linear onde n ^ 1 é um inteiro fixo e encontre os valores das constantes para as quais a integral assume seu valor mínimo. Vamos escrevê-lo com mais detalhes Integrando termo a termo, devido à ortonormalidade do sistema, obtemos Os dois primeiros termos do lado direito da igualdade (7) são independentes, e o terceiro termo é não negativo. Portanto, a integral (*) assume um valor mínimo em ak = sk. A integral é chamada de aproximação quadrática média da função f(x) como uma combinação linear de Tn(x). Assim, a aproximação quadrática média da função /\ assume um valor mínimo quando. quando Tn(x) é a 71ª soma parcial da série de Fourier da função /(x) no sistema (. Fazendo ak = ck, de (7) obtemos a igualdade (9) é chamada de identidade de Bessel. lado é não negativo, então dele segue a desigualdade de Bessel Como i é arbitrário aqui, a desigualdade de Bessel pode ser representada de forma reforçada, ou seja, para qualquer função /, a série de coeficientes de Fourier quadrados dessa função em um sistema ortonormal ) converge . Como o sistema é ortonoral no segmento [-x, r], então a desigualdade (10) traduzida na notação usual da série trigonométrica de Fourier dá a relação do válida para qualquer função f(x) com um quadrado integrável. Se f2(x) é integrável, então devido a Condição necessaria convergência da série no lado esquerdo da desigualdade (11), obtemos isso. Igualdade de Parseval Para alguns sistemas (^n(x)) o sinal de desigualdade na fórmula (10) pode ser substituído (para todas as funções f(x) 6 x) por um sinal de igual. A igualdade resultante é chamada de igualdade de Parseval-Steklov (condição de completude). A identidade de Bessel (9) nos permite escrever a condição (12) em uma forma equivalente pela norma espacial 6]. Definição. Um sistema ortonormal (é chamado completo em b2[ay b] se qualquer função pode ser aproximada com qualquer precisão na média por uma combinação linear da forma com um grande número termos, ou seja, se para qualquer função f(x) ∈ b2[a, b\ e para qualquer e > 0 existe número natural nq e números a\, a2y..., tais que Não O raciocínio acima implica o Teorema 7. Se, por ortonormalização, o sistema ) é completo no espaço, a série de Fourier de qualquer função / neste sistema converge para f(x) na média, ou seja, pela norma Pode-se mostrar que o sistema trigonométrico é completo no espaço, o que implica a afirmação. Teorema 8. Se uma função /0 sua série trigonométrica de Fourier converge para ela na média. 9.5. sistemas fechados. Completude e fechamento de sistemas Definição. Um sistema ortonormal de funções \, é dito fechado se no espaço Li\a, b) não houver nenhuma função ortogonal diferente de zero a todas as funções. No espaço L2\a, b\ os conceitos de completude e fechamento de sistemas ortonormais coincidir. Exercícios 1. Expanda a função na série de Fourier no intervalo (-i-, x) 2. Expanda a função na série de Fourier no intervalo (-r, r) 3. Expanda a função na série de Fourier no intervalo (-r, r) 4. Expanda em uma série de Fourier no intervalo (-jt, r) função 5. Expanda em uma série de Fourier no intervalo (-r, r) a função f (x) \u003d x + x . 6. Expanda em uma série de Fourier no intervalo (-jt, r) a função n 7. Expanda em uma série de Fourier no intervalo (-r, x) a função / (x) \u003d sin2 x. 8. Expanda em uma série de Fourier no intervalo (-m, jt) a função f(x) = y 9. Expanda em uma série de Fourier no intervalo (-mm, -k) a função f(x) = | sinx|. 10. Expanda em uma série de Fourier no intervalo (-x-, r) a função f(x) = g. 11. Expanda em uma série de Fourier no intervalo (-r, r) a função f (x) \u003d sin §. 12. Expanda em uma série de Fourier a função f (x) = n -2x, dada no intervalo (0, x), continuando-a no intervalo (-x, 0): a) de forma par; b) de maneira estranha. 13. Expanda em uma série de Fourier em termos de senos a função / (x) \u003d x2, dada no intervalo (0, x). 14. Expanda em uma série de Fourier a função / (x) \u003d 3-x, dada no intervalo (-2,2). 15. Expanda em uma série de Fourier a função f (x) \u003d |x |, dada no intervalo (-1,1). 16. Expanda em uma série de Fourier em termos de senos a função f (x) \u003d 2x, especificada no intervalo (0,1).